Cancer detection and treatment Flashcards

1
Q

What are the objectives of cancer treatment?and what are the Indications?

A
  • It is important that the goal of treatment is clear in order to manage the expectations of the patient, their relatives and the treating team
  • It is a requirement in the UK that the goal of treatment is recorded in the medical notes of the patient
  • The goal can be either with curative intent or for palliation and there are four main indications for therapy

Indications

Metastatic disease(improve quality life,palliation)

The goal of treatment is an improvement in symptoms with a focus on improving quality of life and survival increments are secondary
As a result the treatment should be well-tolerated and aim to minimise adverse effects
Accept limited toxicity but not at the expense of performance status

Adjuvant therapy(In addition to surgery)

Adjuvant therapy is in addition to another intervention, often surgery (first) that is designed to cytoreduce the tumour bulk and removes all macroscopic disease
Focus is on achieving an improvement in disease-free and overall survival
Intention is on eradicating the micrometastatic disease that remains
Greater toxicity is accepted as the patient is chasing cure of their cancer
May have increased morbidity and mortality associated with the treatment

Primary therapy (neoadjuvant)-Chemo first to reduce tumour size,then surgery)

Primary medical therapy, is where chemotherapy is administered first before a planned cytoreductive procedure (usually surgery)
Can result in a reduced requirement for surgery, increase the likelihood of successful debulking, reduce the duration of hospitalisation and improve the fitness of the patient prior to interval debulking
This approach has the same goals as adjuvant treatment but creates opportunity for translational research to measure responses to treatment and correlate with subsequent specimens removed at the time of surgery.
Most commonly used for an inoperable patient (technical or fitness factors)
Can be used to determine adjuvant goals

Chemoprevention-eg.Tamoxifen

Treatment used in selective at risk groups to prevent the development of cancer
Agents used are to modify risk and improve survival
Only accept very low incidence of toxicity
Many chemotherapy agents are themselves carcinogenic
Impact on overall survival needs careful evaluation

How well did you know this?
1
Not at all
2
3
4
5
Perfectly
2
Q

What are the different treatment approaches?

A

The dosing schedule and interval is determined by the choice of drugs and recovery of the cancer and normal tissues
For most common chemotherapy regimens, the treatment is administered every 21 or 28 days, which defines one cycle
A course of treatment often uses up to 6 cycles of treatment
An increase in effectiveness can be achieved by changing the approach to treatment
In some cases this will increase toxicity too, but it can change the nature of the toxicity and such developments are evaluated in clinical trials

Low-dose therapy is the standard approach and most palliative chemotherapy is given in this manner
The next cycle is started once the bone marrow function has recovered sufficiently to start the treatment (neutrophils >1.0x109/L and platelets >100x109/L)

High-dose therapy uses a higher individual drug dose to achieve a higher cell kill but results in more bone marrow toxicity
This can be minimised by using G-CSF
This approach allows more drug to be delivered within the same schedule of administration but the total received dose can be less than the intended dose due to limitations of non-haematological toxicity

Dose dense therapy involves fractionating the intended dose of drug and administering each fraction on a more frequent basis (often weekly)
Each individual dose produces less toxicity but the anticancer effect is related to the accumulative dose over time
Such an approach can overcome drug resistance, produce a greater cell kill and in some cases produce a response with weekly administration when the 3-weekly schedule demonstrates a lack of response or even disease progression

Alternating therapy involves giving different drugs in an alternating manner
This is most commonly used with haematological malignancies and is designed to treat different subpopulations of cancer cells where individual clones of cells might be resistant to one or more of the agents

How well did you know this?
1
Not at all
2
3
4
5
Perfectly
3
Q

How can toxicity be assessed?

A

Most anticancer treatment is given at the maximum dose tolerated by the patient and adverse effects are inevitable
There are common toxicity criteria (CTC) scales that rate the severity of several hundred side-effects on a four-point scale
This allows an objective measurement and comparison over time throughout treatment
Detailed information and the current version of the CTC is maintained by the National Cancer Institute and is available at http://ctep.cancer.gov/reporting/ctc.html

How well did you know this?
1
Not at all
2
3
4
5
Perfectly
4
Q

How can treatment be evaluated?

A

The evaluation of treatments includes an assessment of overall survival duration, response to treatment, remission rate, disease-free survival and response duration, quality of life, and treatment toxicity
Uniform criteria have been established to measure these, including the response evaluation criteria in solid tumours (RECIST)
This allows clinicians to accurately inform patients of the prognosis, effectiveness and toxicity of chemotherapy and empowers patients to take an active role in treatment decisions

Overall survival rate is the percentage of patients in a study or treatment group that are alive for a certain period of time after the diagnosis of cancer
This is usually stated as a 5-year survival rate, which is the percentage of patients in a defined group that are alive five years after diagnosis or treatment

Remission rate is the percentage of patients that achieve a state where the cancer is no longer detectible
This is complete remission where all signs and symptoms of cancer have disappeared, although cancer still may be in the body
In partial remission, some, but not all, signs and symptoms of cancer have disappeared

Disease-free survival is the length of time after treatment for cancer during which a patient survives with no symptoms or signs of the disease
Disease-free survival may be used in a clinical trial to measure how well a new treatment works

Response is usually assessed by RECIST 1.1 criteria with pre-treatment documentation of target and non-target lesions
All measurable lesions up to a maximum of two lesions per organ and five lesions in total, representative of all involved organs should be identified as target lesions and measured at baseline
Target lesions should be selected on the basis of their size (lesions with the longest diameter) and their suitability for accurate repeated measurements (either by imaging techniques or clinically)
A sum of the longest diameter (LD) for all target lesions will be calculated and recorded as the baseline sum LD, used as a reference to characterise the objective response. All other lesions (or sites of disease) should be identified as non-target lesions and be recorded at baseline

Measurable disease is defined as at least one lesion that can be accurately measured in at least one dimension (longest diameter (LD) to be recorded)
Each lesion must be >10mm when assessed by CT and MRI (if the slice thickness is ≤5mm), and clinical examination; or >20mm when measured by plain X-ray
(For CT and MRI scans where the slice thickness is >5mm, measurability is defined as a lesion with LD > 2x the slice thickness)
For malignant lymph nodes, short axis diameter must by >15mm to be considered pathological and measurable

Non-measurable disease includes aspects that are more subjective in their assessment and based upon factors such as tumour marker levels, presence of ascites or effusions

Residual disease is that left at completion of planned treatment and if present indicates resistance to the treatment and is therefore an adverse prognostic factor
In some circumstances it may be difficult to distinguish residual disease from normal tissue, particularly in patients with post-operative changes following abdominal or pelvic surgery
When the evaluation of complete response depends on this determination, it is recommended that the residual lesion be investigated (fine needle aspirate/biopsy) to confirm the complete response status

How well did you know this?
1
Not at all
2
3
4
5
Perfectly
5
Q

What is the Future of Cancer Chemotherapy?

A

Targeted therapies are now integrated with conventional chemotherapy
Treatment may be optimised to the individual patient
Combinations will be important
Sequencing may be crucial
More targets-not only cancer cells
Rethink the process of screening and clinical trials

How well did you know this?
1
Not at all
2
3
4
5
Perfectly
6
Q

What Investigations can be done for anaplastic carcinoma in cervical nodes

A

CXR; sputum cytology (most reliable in small cell lung cancer)
Thyroid scan + needle biopsy
Nasopharyngeal assessment
Consider diagnosis of undifferentiated lymphoma (exclude with immunophenotyping)

How well did you know this?
1
Not at all
2
3
4
5
Perfectly
7
Q

What Investigations are done for squamous cell carcinoma in inguinal nodes

A

Careful examination of legs, vulva, penis, perineum for primary tumour
Pelvic examination (exclude vaginal/cervical cancer)
Proctoscopy/colposcopy (exclude anal/cervical cancer)

How well did you know this?
1
Not at all
2
3
4
5
Perfectly
8
Q

Which positive immunohistochemical staining on material obtained from a cervical biopsy would imply the presence of human papillomavirus infection?

A

p21

How well did you know this?
1
Not at all
2
3
4
5
Perfectly
9
Q

How common is bowel cancer and what are the risk factors?

A

Colorectal cancer is the third most common cancer and second most common cause of cancer death in the UK. Occurrence is strongly related to age, with 90% of cases occurring in people aged 50 years and over. It is significantly more common in the developed world, occurring most frequently in New Zealand, Canada, USA and UK and lower in Asia, Africa (among blacks), and South America (except Argentina and Uruguay).

Gender: Overall, the incidence of colorectal cancer and mortality rates are higher in men than in women; tumours of the colon are slightly more frequent in women than in men (1.2:1), whereas rectal carcinomas are more common in men than in women (1.7:1).

Increasing age is the most important risk factor for most cancers. Other risk factors for colorectal cancer include the following:

  • Family history of colorectal cancer in a first-degree relative.[[2]
  • Personal history of colorectal adenomas, colorectal cancer, or ovarian cancer.[[3]
  • Hereditary conditions, including familial adenomatous polyposis (FAP) and Lynch syndrome (hereditary nonpolyposis colorectal cancer [HNPCC]).[[6](
  • Personal history of long-standing chronic ulcerative colitis or Crohn colitis.[[7](
  • Excessive alcohol use.[[8]
  • Cigarette smoking.[[9]
  • Race/ethnicity: African American.[[10](
  • Obesity.
How well did you know this?
1
Not at all
2
3
4
5
Perfectly
10
Q

What is the pathogenesis of bowel cancer?

A

Over 70% of all colorectal cancers are adenocarcinomas arising in the mucosa from benign adenomatous polyps. Adenomas are typically slow growing, with a minority progressing to malignancy if they are not surgically removed. This risk of malignant transformation is increased with increasing size of the adenoma and with the length of time it has been present. Once a tumour has become established, it can spread through the layers of the bowel wall and may eventually metastasise to the liver, lungs, bone, brain and skin. Several genetic alterations have been implicated in the progression of benign adenomas to invasive adenocarcinomas.

Approximately 5% of all colorectal cancer cases occur as a consequence of genetic syndromes, the most common of which is hereditary non-polyposis colorectal cancer (HNPCC or Lynch syndrome type I). HNPCC develops as a consequence of germ-line mutations in one of several DNA mismatch repair genes and carries a 40% lifetime risk of developing colorectal cancer. The condition is transmitted in an autosomal dominant pattern. Patients with HNPCC typically develop colorectal cancer in the fourth decade of life. Other malignancies such as ovarian, endometrial, gastric, urinary and hepatobiliary cancer, may occur and these are recognised as Lynch syndrome type II.

Familial adenomatous polyposis (FAP) is a more rare type of hereditary colorectal cancer. It is inherited in an autosomal dominant pattern and arises as a result of germ-line mutations in the tumour suppressor geneAPC.From a young age,affected patients develop numerous benign colonic polyps, which will eventually transform into cancerous lesions, typically in the third and fourth decades of life. Prophylactic colectomy is therefore strongly advised.

The molecular pathogenesis and multistep development of colorectal cancer is covered in the presentation on Carcinogenesis and you should review this if not done already.

Histological types of colon cancer include:

  • Adenocarcinoma - most colon cancers - (mucinous (colloid) adenocarcinoma & signet ring adenocarcinoma
  • Scirrhous tumours
  • Neuroendocrine: Tumours with neuroendocrine differentiation typically have a poorer prognosis than pure adenocarcinoma variants
How well did you know this?
1
Not at all
2
3
4
5
Perfectly
11
Q

How can the features of a left sided bowel cancer be differentiated from those of a right-side?

A

In its early stages, colorectal cancer is often asymptomatic and hence 55% of patients present with advanced disease. Patients with right-sided cancer typically present with a palpable mass in the right iliac fossa, diarrhoea, weight loss, anaemia and occult gastrointestinal bleeding. The clinical features of a left-sided cancer include a palpable mass in the left iliac fossa, change in bowel habit (most commonly constipation), tenesmus, rectal bleeding and signs and symptoms of bowel obstruction. Patients with left-sided cancer typically present earlier than those with right-sided cancer.

How well did you know this?
1
Not at all
2
3
4
5
Perfectly
12
Q

What factor Contributes to the highest risk of Cancer in UC?How can they screened?

A

Pancolitis

Screening-Colonoscopy with multiple biopsies

How well did you know this?
1
Not at all
2
3
4
5
Perfectly
13
Q

What is the most likely explanation for the left iliac fossa mass?
Clinical presentation of bowel cancer?

A

a)Cancer, Diverticulitis

Clinical presentation of bowel cancer?-think and check notion

How well did you know this?
1
Not at all
2
3
4
5
Perfectly
14
Q

What complication can occur with a bowel tumour and If distant sites of disease are present, which sites are most likely?

A

Bowel obstruction, rupture of bowel causing peritonitis

Sites-Liver,kidney,bladder,brain,bone

How well did you know this?
1
Not at all
2
3
4
5
Perfectly
15
Q

What Investigations can be done for bowel cancer and how can they be staged?

A

Laboratory tests

Routine bloods should be performed for all suspected cases. A full blood count (FBC) may show a microcytic anaemia (an iron-deficiency anaemia), as well as LFTs and clotting.

The tumour marker Carcinoembryonic Antigen (CEA) should not be used as a diagnostic test, due to poor sensitivity and specificity, however it is used to monitor disease progression and should be conducted both pre- and post-treatment, screening for recurrence.

Imaging

The gold standard for diagnosis of colorectal cancer is via colonoscopy with biopsy. If a colonoscopy is not suitable for the patient, such as from frailty, co-morbidities, or intolerance, a flexible sigmoidoscopy or CT colonography can be performed for initial diagnosis.

Once the diagnosis is made, several other investigations are required (primarily for staging):

CT scan (Chest/Abdomen/Pelvis) to look for distant metastases and local invasion
A full colonoscopy or CT colonogram is required to check for a 2nd (synchronous) tumour if not used initially
MRI rectum (for rectal cancers only) to assess the depth of invasion and potential need for pre-operative chemotherapy
Endo-anal ultrasound (for early rectal cancers, T1 or T2 only) to assess suitability for trans-anal resection

Teatment decisions can be made with reference to the TNM (tumour, node, metastasis) classification rather than to the older Dukes or the Modified Astler-Coller classification schema.

Dukes’ Staging

Like many other tumours, colorectal cancers are staged according to the TNM system. This stages the cancer according to the depth the tumour invades the bowel wall (T stage), the extent of spread to local lymph nodes (N stage), and whether or not there are distant metastasis (M stage). The Duke’s staging system has now been largely superseded but is still used at some centres for additional staging detail.

Stage Description 5 Year Survival
A Confined beneath the muscularis propria 90%
B Extension through the muscularis propria 65%
C Involvement of regional lymph nodes 30%
D Distant metastasis <10%

How well did you know this?
1
Not at all
2
3
4
5
Perfectly
16
Q

What screening is available for colorectal Ca?What is referral criteria

A

Colorectal Cancer Screening

In the UK, screening is offered every 2 years to men and women aged 60-75 years. For most of the UK, a faecal immunochemistry test (FIT) is used, superseding the faecal occult test, which utilises antibodies against human haemoglobin to detect blood in faeces.

If any of the samples are positive, patients are offered an appointment with a specialist nurse and further investigation via colonoscopy. Since its introduction in 2006, the NHS Bowel Cancer Screening Programme has increased detection of colorectal cancer in people aged 60-69 by 11%

In the UK, NICE guidance recommends that patients should be referred for urgent investigation of suspected bowel cancer if:

≥40yrs with unexplained weight loss and abdominal pain
≥50yrs with unexplained rectal bleeding
≥60yrs with iron‑deficiency anaemia or change in bowel habit
Positive occult blood screening test.

How well did you know this?
1
Not at all
2
3
4
5
Perfectly
17
Q

How can bowel cancer be treated and what is use of CEA?

A

Whilst this patient has a raised serum CEA, this is not diagnostic in isolation but could be used to correlate response to subsequent therapeutic intervention. Whilst microscopic disease cannot be detected using conventional imaging, the high value of the CEA suggests that there may be a greater risk of more widespread microscopic disease but the treatment stratification relies on staging the patient and for that, the primary tumour and nearby lymph nodes need to be surgically removed. Cystic structures in the liver can be a normal variant and MR imaging is best to distinguish the features of metastatic disease from simple cysts. This may be important as pre-operative chemotherapy for metastatic disease and resection of solitary liver metastasis can be considered in selected patients.

All patients should be discussed with the multidisciplinary team (MDT). The only definitive curative option is surgery, although chemotherapy and radiotherapy have an important role as neoadjuvant and adjuvant* treatments, alongside their role in palliation.

Surgical

Surgery is the mainstay of curative management for localised malignancy in the bowel. The general plan in most surgical management plans is suitable regional colectomy, to ensure the removal of the primary tumour with adequate margins and lymphatic drainage, followed either by primary anastomosis or formation of a stoma:

Right Hemicolectomy or Extended Right Hemicolectomy
The surgical approach for caecal tumours or ascending colon tumours, with the extended option performed for any transverse colon tumours. During the procedure the ileocolic, right colic, and right branch of the middle colic vessels (branches of the SMA) are divided and removed with their mesenteries.
Left Hemicolectomy
The surgical approach for descending colon tumours. Similar to the right hemicolectomy, the left branch of the middle colic vessels (branch of SMA/SMV), the inferior mesenteric vein, and the left colic vessels (branches of the IMA/IMV) are divided and removed with their mesenteries.
Sigmoidcolectomy
The surgical approach for sigmoid colon tumours. In this instance, the IMA is fully dissected out with the tumour in order to ensure adequate margins are obtained.
Anterior Resection
The surgical approach for high rectal tumours, typically if >5cm from the anus. This approach is favoured as leaves the rectal sphincter intact if an anastomosis is performed (unlike AP resections). Often a defunctioning loop ileostomy is performed to protect the anastomosis and reduce complications in the event of an anastomotic leak, which can then be reversed electively four to six months later.
Abdominoperineal (AP) Resection
The surgical approach for low rectal tumours, typically <5cm from the anus. This technique involves excision of the distal colon, rectum and anal sphincters, resulting in a permanent colostomy.
*Elective colectomies are often performed laparoscopically, as this offers faster recovery times, reduced surgical site infection risk, and reduced post-operative pain, with no difference in disease recurrence or overall survival rates when compared to open surgery

Hartmann’s Procedure
This procedure is used in emergency bowel surgery, such as bowel obstruction or perforation. This involves a complete resection of the recto-sigmoid colon with the formation of an end colostomy and the closure of the rectal stump

Chemotherapy

Chemotherapy is indicated typically in patients with advanced disease (adjuvant chemotherapy in Dukes’ C colorectal cancer has been found to reduce mortality by 25%).

An example chemotherapy regime for patients with metastatic colorectal cancer is FOLFOX, comprised of Folinic acid, Fluorouracil (5-FU), and Oxaliplatin, which has been demonstrated to significantly improvement in 3-year disease-free survival for patients with advanced colon cancer.

Radiotherapy

Radiotherapy can be used in rectal cancer (it is rarely given in colon cancer due to the risk of damage to the small bowel), most often as neo-adjuvant treatment.

It is of particular use in patients with rectal cancers which look on MRI to have a “threatened” circumferential resection (i.e. within 1mm). They can undergo pre-operative long-course chemo-radiotherapy to shrink the tumour, thereby increasing the chance of complete resection and cure.

Palliative Care

Very advanced colorectal cancers will be managed palliatively, focusing on reducing cancer growth and ensuring adequate symptom control. Whilst a large variety of palliative care options are available to such patients, important surgical options that can offered include:

Endoluminal stenting can be used to relieve acute bowel obstruction in patients with left-sided tumours
The main side-effects of stents are perforation, migration, and incontinence
Stoma formation can be performed for patients with acute obstruction, usually with either a defunctioning stoma or palliative bypass
Resection of secondaries, not commonly performed but can done with adjuvant chemotherapy for any liver metastases

How well did you know this?
1
Not at all
2
3
4
5
Perfectly
18
Q

What are the factors associated with recurrence?

A

the degree of red and processed meat intake before diagnosis was associated with a higher risk of death

Aspirin may reduce risk?check notion

How well did you know this?
1
Not at all
2
3
4
5
Perfectly
19
Q

What are the different imaging techniques used to view the colon?

A

So a colonoscopy examines from the anus all the way to the ceacum, and can also enter the small intestine to view the terminal ileum.

Colonoscopy v flexisigmoidoscopy v rigid sigmoidoscopy:

There are couple of procedures similar to a colonoscopy, however there are key difference

Colonoscopy– views of the full colon, from rectum to terminal ileum

Flexi-sigmoidoscopy– used to view pathology on left side of colon, from the rectum to splenic flexure

Rigid sigmoidoscopy– used to view rectal pathology, from the rectum to approximately 25cm along the bowel. This is less popular now as usually a flexisigmoidoscopy would be done instead

Proctoscopy– used to view the rectum, often to carry out minor haemorrhoid procedures

CT colonography– a specific type of CT scan used if a patient is unable to have colonoscopy, Gas is used to inflate the colon, which allows the bowel wall to be imaged, therefore creating a ‘virtual’ colonoscopy

How well did you know this?
1
Not at all
2
3
4
5
Perfectly
20
Q

What are Indications for colonoscopy?

A

Diagnostic > surveillance > therapeutic (of these diagnostic is the commonest reason)

Bowel cancer screening:The NHS invites all people for bowel screening at the age of 55. This is usually a flexi-sigmoidoscopy rather than a colonoscopy. Although this does occasionally diagnose a cancer, the main purpose is to look and remove polyps, which can develop into a cancer if left untreated.

Most colonoscopies will involve biopsies to be taken. This allows the bowel to be examined from a histopathology point of view, which aides diagnosis and surveillance.

Diagnostic:

Symptoms…

  • PR bleeding / faecal occult blood
  • Change in bowels habit (constipation / diarrhoea, usually for >6 weeks)
  • Weight loss
  • Iron deficient anaemia (microcytic anaemia can be the first sign of a right sided colon cancer)

Commonest differentials…

  • Cancer (if there is a concern that the diagnosis might be a cancer, the patient should be referred via the 2-wee-wait pathway to receive an urgent colonoscopy)
  • Inflammatory bowel disease
  • Diverticular disease

Surveillance:

  • Polyps
  • Previous cancer
  • IBD

Therapeutic:

  • Polypectomy (removal of polyps
  • Colonic stenting (used most commonly in palliative treatment of colorectal cancers)
  • Colonic dilatation (used most commonly in strictures of the colon)
How well did you know this?
1
Not at all
2
3
4
5
Perfectly
21
Q

What are Indications for colonoscopy?

A

Main risks:

  • Pain
  • Bleeding
  • Perforation
  • Requirement for major operation
  • Missing lesion
  • Complications of sedation

These should all be included on a written consent form that is signed by the patient and the endoscopist before the procedure.

How well did you know this?
1
Not at all
2
3
4
5
Perfectly
22
Q

What preparation is needed before colonoscopy?

A

Do a mosler

Medication: certain medications should be stopped before the procedure, these include anticoagulants, medications containing iron, and codeine

Diet: Patients are advised to drink plenty of fluids and have a low fibre diet for 2-3 days beforehand

Bowel prep: usually requires the patient to take strong laxative sachets the day before or on the day of the procedure. This varies slightly between hospitals, but patients are always given clear instructions of how to take it. The commonest types used are citrafleet, picolax, klean-prep, and moviprep

After the procedure:

·The patient is monitored and is usually able to go home after an hour or so

·The images and pathology are reviewed, and then the patient is contacted with the results. Depending on the findings the patient is informed via letter or is brought back to hospital for a face-to-face appointment.

How well did you know this?
1
Not at all
2
3
4
5
Perfectly
23
Q

How common is cancer?

A
  • Cancer represents a significant economic burden for the global economy and is now the third leading cause of death worldwide
  • By 2030, it is projected that there will be 26 million new cancer cases and 17 million cancer deaths per year
  • The developing world is disproportionately affected by cancer and in 2008 developing nations accounted for 56% of new cancer cases and 75% of cancer deaths
How well did you know this?
1
Not at all
2
3
4
5
Perfectly
24
Q

What are the most common cancer associated with Smoking?

A

Tobacco contributes to the development of 3 million cancers (lung, oropharynx, larynx, bladder, kidney), which could be prevented by smoking cessation

How well did you know this?
1
Not at all
2
3
4
5
Perfectly
25
Q

What are the common cancers associated with obesity?

A

Like tobacco, obesity, physical inactivity and poor nutrition are established causes of several types of cancer
Diet contributes to 3 million cancer deaths per year (gastric, colon, oesophagus, breast, liver, oropharynx and prostate) and diet modification could reduce these by avoiding animal fat and red meat, increasing fibre, fresh fruit and vegetable intake, as well as avoiding obesity

How well did you know this?
1
Not at all
2
3
4
5
Perfectly
26
Q

What are challenges for Cancer care?

A
  • Prevention with vaccination against certain cancers could reduce the cancer burden with protection against HBV and HPV
  • Education is important as low rates of literacy are associated with regions of poverty
  • Education about cancer could result in earlier diagnosis, better engagement with screening, and acceptance of diagnostic and treatment services
  • Such approaches need to reflect the local cultural requirements
  • Access to treatment is resource limited as treatment for cancer relies on surgery, radiotherapy and chemotherapy, all of which remain expensive and often unavailable in developing nations
  • New targeted therapies will be too expensive and therefore the newest developments in therapy will be unavailable without successful engagement of the pharmaceutical industry to negotiate reimbursement schemes which might make new drugs more affordable and accessible
  • Cure the curable: with a greater understanding of the hallmarks of cancer, specific features of cancers can be used as targets for treatment and could be used to reclassify the cancers
  • Understanding the microenvironment of the cancer cell is vital to delivering successful future therapies but open access to research findings for all nations should be a key principle for funding research
  • Provide palliation whenever it is required as the majority of cancer treatment is not aimed at cure, more to control symptoms of the patient
  • Access to analgesia is often poor with only 9% of the world’s morphine used in developing nations which have 83% of the world’s population
  • In some regions of Africa, patients have to walk for more than a day in each direction to and from a pharmacy to receive only 5 days supply of medication
  • There are persisting misconceptions about the problems of strong opioid analgesia that have yet to be overcome
  • End of life care is not expensive but requires involvement of the family and other care givers
  • It can be improved by access to better training and education and provision of community-based services that understand the diversity and requirements of the local population
How well did you know this?
1
Not at all
2
3
4
5
Perfectly
27
Q

What is performance status and why is it checked?

A

§ One of the most important factors that impacts on the planning of treatment and prognosis is the performance status of the patient
§ This requires an assessment of their functional capacity, ability to self-care and mobility
§ The performance status correlates with prognosis and tolerance of treatment and a number of different scales are used; the more common being the ECOG and Karnofsky scales
§ Patients with performance status 3 or 4 do not tolerate treatment as well and indeed some systemic chemotherapy may shorten their life

How well did you know this?
1
Not at all
2
3
4
5
Perfectly
28
Q

What cancers can cause lymphadenopathy in Supraclavicular fossa?

A

When considering a patient with lymphadenopathy in the supraclavicular fossa or axilla, consider the anatomical sites that drain to those locations. In this case, breast, lung, oesophagus and stomach.

How well did you know this?
1
Not at all
2
3
4
5
Perfectly
29
Q

How to identify the primary cancer in a patient presenting with metastatic disease?How can it be managed?

A

For most patients who present with metastatic disease, routine examination and investigation will quickly disclose the underlying primary tumour. However, for 1-5% of patients, the primary site remains undisclosed because it is too small to be detected or has regressed. The usual histological diagnosis in these patients with unknown primary site is adenocarcinoma or poorly differentiated carcinoma (see case 7).

Important considerations:

  • What is the rationale for establishing primary tumour site?
  • Diagnosing treatable disease (see table below)
  • Avoiding over treating unresponsive disease (iatrogenic morbidity in resistant disease)
  • Preventing complications related to occult primary, e.g. bowel obstruction, pathologic fracture
  • Prognostic clarification

The most important first investigation to aid diagnosis is to biopsy or excise the lymph node in the neck. Histology of the lymph node in this case demonstrated squamous cell carcinoma.

“Five highly treatable subsets of unknown primary site have been identified which have more favourable outcomes and require distinct management.”

How well did you know this?
1
Not at all
2
3
4
5
Perfectly
30
Q

What are the risk factors for cervical cancer?

A

“Increasing age is the most important risk factor for most cancers. The primary risk factor for cervical cancer is human papillomavirus (HPV) infection.

Other risk factors for cervical cancer include the following:

  • High parity and HPV infection
  • Smoking cigarettes and HPV infection
  • Long-term use of oral contraceptives and HPV infection
  • Immunosuppression
  • Having first sexual encounter at a young age
  • High number of sexual partners
  • Exposure to diethylstilbestrol (DES)in utero

Human papillomavirus (HPV) infection

HPV infection is a necessary step in the development of virtually all precancerous and cancerous lesions. Epidemiologic studies convincingly demonstrate that the major risk factor for development of preinvasive or invasive carcinoma of the cervix is HPV infection, far outweighing other known risk factors.”1”Transient HPV infection is common, particularly in young women, while cervical cancer is rare. The persistence of an HPV infection leads to increased risk of developing precancerous and cancerous lesions.

The strain of HPV infection is also important in conferring risk. There are multiple subtypes of HPV that infect humans; of these, subtypes 16 and 18 have been most closely associated with high-grade dysplasia and cancer. Studies suggest that acute infection with HPV types 16 and 18 conferred an 11-fold to 16.9-fold risk of rapid development of high-grade CIN. Further studies have shown that infection with either HPV 16 or 18 is more predictive than cytological screening of high-grade CIN or greater disease, and that the predictive ability is seen for up to 18 years after the initial test.

There are two commercially available vaccines that target anogenital-related strains of HPV. The vaccines are directed towards HPV-naïve girls and young women, and although penetration of the vaccine has been moderate, significant decreases in HPV-related diseases have been documented.”1

How well did you know this?
1
Not at all
2
3
4
5
Perfectly
31
Q

What are the clinical features and prognostic factors?

A

“Early cervical cancer may not cause noticeable signs or symptoms. Possible signs and symptoms of cervical cancer include:

  • Vaginal bleeding
  • Unusual vaginal discharge
  • Pelvic pain
  • Dyspareunia
  • Postcoital bleeding”1

Prognostic factors

  • “HIV status: Women with HIV have more aggressive and advanced disease and a poorer prognosis
  • C-mycoverexpression: A study of patients with known invasive squamous carcinoma of the cervix found that overexpression of the C-myconcogene was associated with a poorer prognosis
  • Number of cells in S phase: The number of cells in S phase may also have prognostic significance in early cervical carcinoma
  • HPV-18 DNA: This has been found to be an independent adverse molecular prognostic factor. Two studies have shown a worse outcome when HPV-18 was identified in cervical cancers of patients undergoing radical hysterectomy and pelvic lymphadenectomy
  • A polymorphism in the Gamma-glutamyl hydrolase enzyme, which is related to folate metabolism, has been shown to decrease response to cisplatin, and as a result is associated with poorer outcomes.”
How well did you know this?
1
Not at all
2
3
4
5
Perfectly
32
Q

What are the poor prognostic factors associated with adenocarcinoma?

A

Poor prognosis is associated with adenocarcinoma, lymph node involvement, advanced clinical stage, large primary tumour and early recurrence. Relapse after 5 years is unusual.

How well did you know this?
1
Not at all
2
3
4
5
Perfectly
33
Q

How can cervical cancer be treated?

A

Optimum treatment is determined by the stage of the disease, age and general health of the woman and plans for future fertility.

  • Local excision using loop diathermy is performed for CIN 2/3 confined to the visible ectocervix
  • Loop biopsy is performed for CIN 3 with disease extending into cervical canal
  • Simple hysterectomy is performed for micro-invasive disease
  • Stage IB or 2 cervical cancers are treated by radical hysterectomy with pelvic lymphadenectomy or pelvic radiotherapy. Both methods are equally effective
  • Stage 2B and 3 should be treated with pelvic radiotherapy and patients treated with curative intent typically receive chemoradiotherapy with cisplatin as a radiation sensitizer
  • Stage 4 and recurrent disease are treated with chemotherapy. Radiotherapy can be used to treat specific site of metastasis.
  • Chemotherapy alone has no role in the adjuvant treatment of cervical cancer
How well did you know this?
1
Not at all
2
3
4
5
Perfectly
34
Q

What Investigation are done for Suspected cervical cancer and how it be staged?

A

n a woman presenting with symptoms suggestive of cervical cancer, the initial investigation depends on age:

Pre-menopausal – test for chlamydia trachomatis infection
If positive; treat for chlamydia infection. If symptoms persist after treatment, refer for colposcopy and biopsy.
If negative; a colposcopy and biopsy is usually performed.
Post-menopausal – urgent colposcopy and biopsy.
A colposcopy is where a colposcope (modified microscope) is used to produce a magnified view of the cervix. Acetic acid is used to stain dysplastic areas, and a biopsy is taken.

If the diagnosis of cervical cancer is confirmed, further investigations are required:

Basic blood tests – such as full blood count, liver function tests and urea & electrolytes
CT Chest-Abdomen-Pelvis – looking for metastases.
Further staging scans – e.g. MRI pelvis, PET.
+/- examination under anaesthesia with further biopsies.

The International Federation of Gynaecology and Obstetrics (FIGO) staging system is used for cervical cancer:

Stage 0 – Carcinoma in-situ
Stage 1 – Confined to cervix
A) Identified only microscopically.
B) Gross lesions, clinically identifiable.
Stage 2 – Beyond cervix but not pelvic sidewall/ involves vagina but not lower 1/3
A) No parametrial involvement.
B) Obvious parametrial involvement.
Stage 3 – Extends to pelvic sidewall/ involves lower 1/3 vagina/ hydropnephrosis not explained by another cause.
A) No extension to sidewall.
B) Extension to sidewall and/or hydronephrosis.
Stage 4 – Extends to bladder or rectum, or metastases
A) Involves bladder/rectum.
B) Involves distant organs

How well did you know this?
1
Not at all
2
3
4
5
Perfectly
35
Q

How is screening done for Cervical cancer?

A

Cervical cancer is very rare in women younger than 25. But changes in the cells of the cervix are quite common in this age group. These changes often return to normal and are less likely to develop into cancer

Using Cervical smear test

How well did you know this?
1
Not at all
2
3
4
5
Perfectly
36
Q

How may Lung cancer present?

A
  • A small proportion of patients will have no presenting symptoms
  • Most patients will present with intrathoracic symptoms (e.g. cough, dyspnoea, chest pain, haemoptysis, recurrent chest infections)
  • Dysphagia may result from extrinsic compression by the primary tumour
  • A hoarse voice suggests invasion of the recurrent laryngeal nerve
  • Extrathoracic symptoms may include anorexia, weight loss, malaise and lethargy
How well did you know this?
1
Not at all
2
3
4
5
Perfectly
37
Q

What are the most typical clinical features of Lung Cancer?

A
  • Hands: nicotine staining, clubbing
  • Neck: lymphadenopathy
  • Chest: signs of pulmonary collapse or consolidation
  • Superior vena caval obstruction
  • Pancoast’s syndrome (Chest wall pain, Horner’s syndrome (miosis, anhydrosis, partial ptosis and enophthalmos) and pain in T1 dermatomal distribution due to apical tumour invading chest wall, interrupting sympathetic chain and invading T1 nerve root)
  • Non-metastatic manifestations: haematological (e.g. anaemia), neuromuscular (e.g. Eaton-Lambert syndrome), cutaneous (e.g. acanthosis nigricans), due to ectopic hormone production (e.g. hypercalcaemia), etc
How well did you know this?
1
Not at all
2
3
4
5
Perfectly
38
Q

What is the Epidemiology of Lung Cancer?

A
  • Lung cancer is the commonest malignant solid tumour in the UK (excluding non-melanomatous skin cancer)
  • Causes approximately 35,000 deaths per year in the UK
  • In the UK 7% of patients are alive and disease-free 5 years from diagnosis
  • The incidence of lung cancer is approximately 42,000 cases per year in the UK
  • Peak incidence is at 60-70 years
  • The incidence has paralleled trends in cigarette smoking after a 20- to 50-year lag period
  • Among the major histological subtypes of lung cancer, the association between the extent of tobacco exposure and risk is particularly strong for squamous cell and SCLC (small-cell lung cancer)
  • Male to female ratio is approximately 3:1
  • In males the incidence is decreasing, in females it is increasing
  • Incidence is greater than prevalence
How well did you know this?
1
Not at all
2
3
4
5
Perfectly
39
Q

What are the different types of Lung cancer?

A
  • Spread is circumferential and longitudinal along the bronchus of origin
  • Tumours frequently involve regional lymphatics
    • Spread is to ipsilateral peribronchial and hilar nodes, followed by mediastinal, contralateral hilar and supraclavicular nodes
  • There is a propensity to disseminate widely via the bloodstream and virtually any site may be involved

Small cell lung cancer(SCLC) accounts for 20% of all lung cancers and arises in the larger airways and tends to be a more central tumour. Most patients present with systemic disease and it frequently metastasises, via haematogenous spread, to the liver, skeleton, bone marrow, brain and adrenal glands. The small cells contain dense neurosecretary granules which can produce ectopic biological substances resulting in Cushing’s syndrome (ACTH) and SIAD. Mutations inRB1andTP53are found in 80% of patients with SCLC and abnormal DNA methylation of the cyclin D2 gene is common.

Non-small cell lung cancers(NSCLC) (80% of lung cancers) arise from the epithelial cells of the lung from the central bronchi to the terminal alveoli. It can be divided into 3 main types:

  • Squamous cell carcinoma (50%) is the commonest histological diagnosis, often presenting as an obstructive lesion of the bronchus causing infection. They can cavitate on a chest radiograph and tend to grow slowly, spreading locally and disseminating late
  • Adenocarcinoma (15%) arises from the bronchial mucosal glands, and tends to occur in the periphery. As such findings can represent a metastasis from a distant site, careful patient assessment is required. There is less association with smoking, but they can originate in scar tissue and carry a high risk of metastatic spread, often to mediastinal lymph nodes and pleura producing an effusion
  • Large cell carcinoma (10%) often presents as a large peripheral mass on a chest radiograph and can have neurosecretory elements that produce paraneoplastic features. They tend to be poorly differentiated, can grow rapidly and metastasise early

Genetics

There is a 2.5-fold increased risk of lung cancer where there is a significant family history, despite their own smoking history and rarely lung cancers develop in patients with germ line mutations in genes such as inRbandTP53.

How well did you know this?
1
Not at all
2
3
4
5
Perfectly
40
Q

What are the risk factors for Lung Cancer?

A

“Increasing age is the most important risk factor for most cancers but other risk factors for lung cancer include:

  • History of or current tobacco use: cigarettes, pipes, and cigars
  • Exposure to cancer-causing substances in secondhand smoke
  • Occupational exposure to asbestos, arsenic, chromium, beryllium, nickel, and other agents
  • Radiation exposure from any of the following:
  • Radiation therapy to the breast or chest
  • Radon exposure in the home or workplace
  • Medical imaging tests, such as computed tomography (CT) scans
  • Atomic bomb radiation
  • Living in an area with air pollution
  • Family history of lung cancer
  • Human immunodeficiency virus infection
  • Beta carotene supplements in heavy smokers

The single most important risk factor for the development of lung cancer is smoking. For smokers, the risk for lung cancer is on average tenfold higher than in lifetime non-smokers (defined as a person who has smoked <100 cigarettes in his or her lifetime). The risk increases with the quantity of cigarettes, duration of smoking, and starting age.

Smoking cessation results in a decrease in precancerous lesions and a reduction in the risk of developing lung cancer. Former smokers continue to have an elevated risk of lung cancer for years after quitting. Asbestos exposure may exert a synergistic effect of cigarette smoking on the lung cancer risk.”1

How well did you know this?
1
Not at all
2
3
4
5
Perfectly
41
Q

What Investigations are done in Lung Cancer?

A

The aim of investigation is to gain a histological diagnosis and determine the extent of spread (stage), in order to determine the most appropriate treatment plan.

All patients with known or suspected lung cancer should have:

  • History
  • Examination
  • CXR
  • FBC
  • Biochemical profile (including U&Es, LFTs, LDH and serum calcium)

Sputum cytology can be unreliable and a biopsy is preferable. CT imaging is used to assess the tumour size and spread, determine lymph node involvement or identify chest wall invasion or metastasis to other sites.

Tissue for diagnostic testing is usually obtained during a bronchosocopy but a fine needle aspiration sample from an involved lymph node or CT-guided transthoracic biopsy may be considered. Fluid cytology from a pleural effusion can be useful if present at diagnosis. If bone pain is present, plain X-rays and bone scan imaging are required to exclude metastasis.

MRI scan of the chest can determine lymph node involvement but PET/CT is increasingly being used as an alternative to an invasive mediastinoscopy to determine operability of the patient.

Patients with small cell lung cancer should have imaging of the upper abdomen, as the cancer can often spread to the liver and adrenal glands

Patients with non-small cell lung cancer being considered for radical treatment may require:

  • pulmonary function tests
  • contrast CT scan of the thorax and upper abdomen
  • V/Q scan
  • MRI scan
  • ultrasound scan
  • mediastinoscopy, laryngoscopy, PET-CT
  • additional investigation if clinical, biochemical or radiological suspicion of metastatic disease.
How well did you know this?
1
Not at all
2
3
4
5
Perfectly
42
Q

What are the Prognostic factors?

A

Multiple studies have attempted to identify the prognostic importance of a variety of clinicopathologic factors and those that correlate with adverse prognosis include:

Presence of pulmonary symptoms

  • Large tumor size (>3 cm)
  • Nonsquamous histology
  • Metastases to multiple lymph nodes within a TNM-defined nodal station
  • Vascular invasion

For patients with inoperable disease, prognosis is adversely affected by poor performance status and weight loss of more than 10%. These patients have been excluded from clinical trials evaluating aggressive multimodality interventions.

How well did you know this?
1
Not at all
2
3
4
5
Perfectly
43
Q

What are the complications of Lung cancer?

A

Disease-related complications in NSCLC

  • Local invasion
    • SVC obstruction
    • Pleural effusion
  • Distant metastasis
    • Brain, liver, bone
  • Non-metastatic
    • Hypercalcaemia
    • Cushing syndrome
    • SIAD
    • Neurological syndromes
How well did you know this?
1
Not at all
2
3
4
5
Perfectly
44
Q

How does SVC present?

A

SVC obstruction

  • SVC syndrome is the clinical expression of obstruction of blood flow through the SVC
  • Characteristic symptoms and signs may develop quickly or gradually
  • Caused by compression, invasion, or thrombosis in the superior mediastinum
  • Symptoms and signs may be aggravated by bending forward, stooping, or by lying down
  • Treatment is directed at the underlying cause
  • Prognosis of patients with SVC syndrome strongly correlates with the prognosis of the underlying disease

Common symptoms and signs of SVC obstruction

Dyspnoea
Facial swelling
Head fullness
Cough
Arm swelling
Chest pain

Primary pathologic diagnoses for SVC Obstruction

Lung cancer
Lymphoma
Other malignancies (Primary or secondary)
Nonneoplastic
Undiagnosed

Chest X-ray findings of SVC obstruction

Superior mediastinal widening
Pleural effusion
Right hilar mass
Bilateral diffuse infiltrates
Cardiomegaly
Calcified paratracheal nodes
Anterior mediastinal mass
Normal
How well did you know this?
1
Not at all
2
3
4
5
Perfectly
45
Q

How can brain mets present?how can they be managed?

A

Increasingly diagnosed

  • Patients living longer
  • Better imaging modalities

Frequency

  • Lung (50%)
  • Breast (15-20%)
  • CUP (10%)
  • Melanoma (10%)
  • GI (5%)

Presenting features of brain metastasis

  • Headache (70-80%)
  • Cognitive dysfunction (40%)
  • Neurological deficit (40%)
  • Seizures (15-20%)

Treatment for brain metastasis

Steroids: first described in 1961, can:

  • decrease peritumoral oedema
  • minimal mineralocortocoid activity
  • can double overall survival
  • no single optimal dose or schedule known, but 10 mg stat iv, followed by 16 mg per 24 hours is a generally accepted

Anticonvulsants:

  • choice of IV or PO: tailor it to the patients needs
  • traditionally phenytoin is the agent of choice, but many centres use sodium valproate, although there are problems with the enzyme inducing effect and erythema multiforme
  • No evidence for prophylactic anticonvulsive therapy

Surgery:

  • Evidence is limited - Series are usually small and often retrospective
  • Role of surgery clear for solitary brain mets or singular brain mets with quiescent extra cranial disease
  • Generally, post surgical Whole Brain Irradiation recommended

Radiotherapy:

  • RT has been postulated to increase survival, however role of XRT in poor PS patients remains undefined and there is no RCT against best supportive care
  • No optimal dose schedule have been established, but 30Gy/10# or 20Gy/4-5# is quite normal

Chemotherapy:

  • Role of chemotherapy is generally disappointing, due to inability for chemotherapy to pass blood brain barrier despite disease
  • More role in germ cell and small cell tumours as these are particularly sensitive to treatment
How well did you know this?
1
Not at all
2
3
4
5
Perfectly
46
Q

What is cellular communication and control?and how does What is cellular communication and control?

A

Signalling at the cell surface
Signalling pathways that control gene activity
Integration of signals and gene controls
Regulating the cell cycle
Cell birth, development and death
Mean of communication and coordination for cancer cells to exploit and use this for their advantage

Cell signalling is part of a complex system of communication that governs basic cellular activity and coordinates cell actions
Used in development, tissue repair and immunity
Essential for normal tissue homeostasis
Errors cause cancer, autoimmunity and diabetes

Signal transduction-Occurs when an extracellular signalling molecule binds to and activates a cell surface receptor
The receptor alters intracellular molecules creating a response
A second messenger transmits the signal into the cell, eliciting a physiological response

How well did you know this?
1
Not at all
2
3
4
5
Perfectly
47
Q

How does Hormonal signalling work?

A

Induce a specific response or alter the activities of the target tissue
Act specifically via receptors located on, or in, target tissue
A substance may be released by one cell and recognized by different target cells producing heterogenous responses

Endocrine signalling
Paracrine signalling
Autocrine signalling
Contact-dependent signalling (juxtacrine)

Same signal, different response
Different combinations, different response

How well did you know this?
1
Not at all
2
3
4
5
Perfectly
48
Q

What factors determine Specificity of biosignalling ?

A

Molecular complementarity between signal and receptor
multiple non-covalent interactions similar to substrate-enzyme, solute-transporter, and antigen-antibody interactions
Cell-specific expression of receptors
only cells with receptors specific for the signal can respond
Cell-specific expression of signal transduction proteins
same signal-receptor may activate or inhibit depending on other signal transduction proteins present
Cell-specific expression of effector proteins
differential response of liver, skeletal muscle and adipose cells to epinephrine depends on expressed enzymes

How well did you know this?
1
Not at all
2
3
4
5
Perfectly
49
Q

What are second and third signals?

A

Small molecules synthesised in cells in response to an external signal and are responsible for the intracellular signal transduction

Ca2+			ions
DG, ceramide	lipid derivatives
IP3			carbohydrate derivatives
cAMP, cGMP	nucleotides
Ras, JAK, Raf	proteins

Third messengers are the molecules which transmit message from outside to inside of nucleus or from inside to outside of nucleus
Also called DNA binding protein

How well did you know this?
1
Not at all
2
3
4
5
Perfectly
50
Q

What are the 4 mechanism of signal transduction pathways?

A

There are 4 types of mechanism
1. Direct ligand-gated channel type
Nicotinic acetylcholine, GABA
receptors, neurotransmitters

Acts as a gate when the receptor changes shape
When a signal molecule binds as a ligand to the receptor, the gate allows specific ions, such as Na+ or Ca2+ through a channel in the receptor

  1. G-protein-coupled type
    Muscarinic acetylcholine, adrenergic receptors
    Constitute a large protein family of receptors that are also called seven-transmembrane receptors because they pass through the cell membrane seven times
    Found only in eukaryotes, including yeast, choanoflagellates and animals
    The ligands that bind and activate these receptors include light-sensitive compounds, odours, pheromones, hormones and neurotransmitters, varying in size from small molecules to peptides and large proteins
    Involved in many diseases and are the target of approximately 40% of all modern medicinal drugs such as Hydrocodone and Lisinopril
  2. Tyrosine kinase-linked type
    Epidermal growth factor, VEGF
  3. Nuclear receptor super family
    Ligand-sensitive transcription factor
    Steroid and thyroid hormones
    Intracellular receptor proteins are found in the cytosol or nucleus of target cells
    Small or hydrophobic chemical messengers can readily cross the membrane and activate receptors
    Examples of hydrophobic messengers are the steroid and thyroid hormones
    An activated hormone-receptor complex can act as a transcription factor, turning on specific genes
How well did you know this?
1
Not at all
2
3
4
5
Perfectly
51
Q

What physiological processes are controlled by G-protein coupled receptors?

A

Two principal signal transduction pathways involve the G protein–coupled receptors
cAMP signal pathway
phosphatidylinositol signal pathway
The human genome encodes thousands of G protein-coupled receptors, about 350 of which detect hormones, growth factors, and other endogenous ligands
Approximately 150 of the GPCRs found in the human genome have unknown functions

Involved in a variety of physiological processes:

Vision: The opsins use a photoisomerization reaction to translate electromagnetic radiation into cellular signals
Taste: GPCRs in taste cells mediate release of gustducin in response to bitter- and sweet-tasting substances
Smell: Receptors of the olfactory epithelium bind odorants (olfactory receptors) and pheromones (vomeronasal receptors)
Behavioural and mood regulation: Receptors in the mammalian brain bind several different neurotransmitters, including serotonin, dopamine, GABA, and glutamate
Homeostasis modulation: e.g., water balance

Inflammation: Chemokine receptors bind ligands that mediate intercellular communication between cells of the immune system; receptors such as histamine receptors bind inflammatory mediators and engage target cell types in the inflammatory response

Immune system regulation: involved in suppression of TLR-induced immune responses from T cells.

Autonomic nervous system transmission: Both the sympathetic and parasympathetic nervous systems are regulated by GPCR pathways, responsible for control of many automatic functions of the body such as blood pressure, heart rate, and digestive processes

Cell density sensing: A novel GPCR role in regulating cell density sensing

How well did you know this?
1
Not at all
2
3
4
5
Perfectly
52
Q

What is the role of EGFR in human cancers?

A

EGFR can influence the regulation of tumour cell cycle progression, repair, and survival, and is involved in tumour metastasis
Binding of specific ligands to EGFR (e.g. EGF, TGF-) activates the receptor and triggers signal transduction cascades that affect cell proliferation
Many human cancers express EGFR on the cell surface
Inhibition of EGFR on tumour cells may inhibit the growth or progression of EGFR-positive tumours
Many point mutations are recognised within EGFR and these correlate with response to therapy

Development of monoclonal Abs

Binds to the extracellular domain of the EGFR
Blocks growth factor binding and signal transduction
Combined treatment of mAbs with cytotoxic drugs increases anti-tumour activity in tumour xenograft models
Development of recombinant humanized and chimeric mouse-human antibodies
Examples:
trastuzumab
cetuximab

Tyrosine kinase inhibitor
Binds to the intracellular tyrosine kinase domain and inhibits tyrosine autophosphorylation and downstream intracellular signalling
Compete with ATP for binding with the intracellular catalytic domain of TK
Examples:
gefitinib
erlotinib

Immunoconjungates

Binds to the extracellular domain of the receptor
Activate receptors and when internalised the isotope or toxin kills the cell
Examples:
Rhenium186 anti-EGFR Ab h-R3
Pseudomonas exotoxin A

Antisense approach
Binds to EGFR or ligand mRNA
Prevents transcription and receptor or ligand production

How well did you know this?
1
Not at all
2
3
4
5
Perfectly
53
Q

What are mechanisms of EGFR activation in tumour cells?

A

1) Overexpression of EGFR protein
2) Increase autocrine ligand loop
3) heterodimerisation and cross-talk
4) reduce phosphatase
5) mutant EGFR

How well did you know this?
1
Not at all
2
3
4
5
Perfectly
54
Q

What is Carcinogenesis?

A

The process by which normal cells are transformed into cancer

It is clear that changes in the host genome are the final common pathway in the process of carcinogenesis, whatever the initial aetiology

A multi-step process resulting from the accumulation of errors in vital regulatory pathways
Can be initiated in a single cell which then multiplies and acquires additional changes providing a survival advantage over its neighbours
Altered cells must be amplified to generate billions of cells that constitute a cancer
Therefore as this takes time, the longer one lives, the more likely one is to develop cancer
The smallest detectable tumour is approximately 1cm in diameter and already contains 1 billion (1,000,000,000) cells

How well did you know this?
1
Not at all
2
3
4
5
Perfectly
55
Q

What is contact inhibition?

A

Contact inhibition is recognised by cells following reproduction and so switch off division (due to the presence of glycoprotein on the cell surface)

How well did you know this?
1
Not at all
2
3
4
5
Perfectly
56
Q

What is the aetiology of Carcinogenesis?

A
Environmental factors
Chemicals and occupational hazards
Physical (radiation)
Ionising radiation
Ultraviolet radiation
Biological
Viruses
Bacteria
Parasitic infections
Drugs and hormones
Nutritional and lifestyle
Genetic factors
How well did you know this?
1
Not at all
2
3
4
5
Perfectly
57
Q

How can hormones cause Carcinogenesis?

A

Cancer can be induced by overproduction of endogenous hormones as well as exogenous substances as contained within the combined oral contraceptive pill (COCP) and hormone replacement therapy (HRT)
Risk of breast cancer is related to the duration of exposure to oestrogens, with risk increased by low parity, early menarche, late menopause and prolonged exposure to oestrogens by the use of HRT
The COCP does not increase risk as it is used at a time in the life cycle when oestrogen is naturally present
Ovarian cancer is related to the number of ovulations; therefore the risk is increased by nulliparity but reduced by the COCP, reducing by 50% in those taking the COCP for 10 or more years
The risk of endometrial cancer in postmenopausal women is increased by using oestrogen-only HRT or tamoxifen

How well did you know this?
1
Not at all
2
3
4
5
Perfectly
58
Q

What is Li Fraumeni syndrome and how is it inherited?

A
A rare syndrome characterised by
premenopausal breast cancer
childhood sarcoma
brain tumours
leukaemia and lymphoma
adrenocortical carcinoma
Associated with a germline mutations in the TP53 gene on chromosome 17p
Inheritance is autosomal dominant with a penetrance of at least 50% by age 50
How well did you know this?
1
Not at all
2
3
4
5
Perfectly
59
Q

What is the significance of tumor microenvironment?

A

cancer cells only make up 40-50% of

Other cells involved-Endothelial cells
pericytes
fibroblast
Lymphocytes
mast cells, macrophage,neutrophil
Basement membrane ECM

These cells are used used as targets for treatment of tumour

60
Q

What are the 10 hallmarks of Cancer and what drugs can be targeted at each stage?

A

1)Sustained proliferative signalling-EGFR inhibitors
monoclonal abs-rastuzumab,cetuximab
Tyrosine kinase inhibitors gefitinib,erlotinib
Immune conjugates and antisense

Ligands
interferons, erythropoietin, growth hormone, other cytokines, some interleukins
Receptors
Conserved multi β strand fold in extracellular domain
JAK kinase associated with cytosolic domain
Cross-phosphorylation of JAK kinases on activation
Signal transduction
Direct activation of cytosolic STAT transcription factors
Ras–MAP kinase pathway
IP3/DAG pathway
PI-3 kinase pathway

2)Evading growth suppressors-Cyclin dependant kinase inhibitors
combretastatin A4

Cancer cells must circumvent powerful programs that negatively regulate cell proliferation
Many of these programs depend on the actions of tumor suppressor genes
Two prototypical tumour suppressors encode the pRb and p53 proteins
Loss of contact inhibition
Corruption of the TGF-ß pathway promotes cancer
Evading growth suppression is more elaborate than simple shutdown of the antiproliferative signaling circuitry

3)Resisting cell death-Proapoptic BH3 mimetic

Cell death is regulated in two fundamentally different ways:

Survival signals to stay alive
Absence of signals called trophic factors, activates a cellular suicide program
Specifically murdered cells
Cell that are damage by injury, mechanical damage or exposure to toxic chemicals

Mechanisms-Necrosis,autophagy,apoptosis,fas death licences,p53 dysfunction

4)Enabling replicative mortality-Telomerase inhibitors

Most normal cell lineages are able to pass through only a limited number of successive cell growth-division cycles
This limitation has been associated with two distinct barriers to proliferation
senescence: a typically irreversible entrance into a nonproliferative but viable state
crisis: in which the majority of cells in the population die
Cancer cells are able to proliferate in culture without evidence of either senescence or crisis (immortalisation)
Telomeres protecting the ends of chromosomes are centrally involved in the capability for unlimited proliferation
Telomerase is a specialised DNA polymerase that adds telomere repeat segments to the ends of telomeric DNA
It is almost absent in nonimmortalised cells but expressed at functionally significant levels in the vast majority (>90%) of immortalised cells, including human cancer cells
Expression correlates with resistance to induction of both senescence and crisis or apoptosis
Suppression of telomerase activity leads to telomere shortening and to activation of one or the other of proliferative barriers
Telomestatin is a telomerase inhibitor in clinical trial

5)Sustained angiogenesis-Inhibitors of VEGF signalling
(bevazizumab)
Tumors require sustenance in the form of nutrients and oxygen as well as an ability to evacuate metabolic wastes and carbon dioxide
The process of angiogenesis, addresses these needs
The normal vasculature becomes largely quiescent. In the adult, as part of physiologic processes such as wound healing and female reproductive cycling, angiogenesis is turned on but only transiently

6)Deregulated cellular energetics-Aerobic glycolysis inhibitors

Under aerobic conditions, normal cells process glucose, first to pyruvate via glycolysis in the cytosol and thereafter to carbon dioxide in the mitochondria
Under anaerobic conditions, glycolysis is favoured to produce ATP

Aerobic glycolysis
Cancer cells can reprogram their glucose metabolism aerobic glycolysis
Upregulating glucose transporters, notably GLUT1, which substantially increases glucose import into the cytoplasm
18-fold lower efficiency of ATP production afforded by glycolysis relative to mitochondrial oxidative phosphorylation
Increased glycolysis allows the diversion of glycolytic intermediates into various biosynthetic pathways, including those generating nucleosides and amino acids
Some tumors have been found to contain two subpopulations of cancer cells that differ in their energy-generating pathways

7)Genomic instability and mutation-PARP inhibitors

Multiple hallmarks depends in large part on a succession of alterations in the genomes of cancer cells
Multistep tumour progression can be portrayed as a succession of clonal expansions, each of which is triggered by the acquisition of an enabling mutant genotype

Cancer cells have increased rates of mutation
The accumulation of mutations can be accelerated by compromising the surveillance systems that normally monitor genomic integrity and force genetically damaged cells into either senescence or apoptosis
The role of TP53 is central guardian of the genome
Key tasks for all cells
Detect DNA damage and activating the repair machinery
Directly repairing the damaged DNA
Inactivating or intercepting mutagenic molecules before they have damaged the DNA

PARP1 is a protein that is important for repairing single-strand breaks (‘nicks’ in the DNA)
If such nicks persist unrepaired until DNA is replicated (which must precede cell division), then the replication itself can cause double-strand breaks to form
Drugs that inhibit PARP1 cause multiple double-strand breaks to form, and in tumours with BRCA1, BRCA2 or PALB2 mutations these double-strand breaks cannot be efficiently repaired, leading to the death of the cells
Normal cells that do not replicate their DNA as often as cancer cells and that lacks any mutated BRCA1 or BRCA2 still have homologous repair operating, which allows them to survive the inhibition of PARP

8)Avoiding Immune destruction-Immune activating anti-CTLA4 mAbs

The immune system operates as a significant barrier to tumour formation and progression
Deficiencies in the development or function of CD8+ cytotoxic T-lymphocytes (CTLs), CD4+ Th1 helper T-cells, or natural killer (NK) cells each led to demonstrable increases in tumour incidence
Highly immunogenic cancer cells may well evade immune destruction by disabling components of the immune system
Recruitment of inflammatory cells that are actively immunosuppressive, including regulatory T-cells (Tregs) and myeloid-derived suppressor cells (MDSCs)
Both can suppress the actions of cytotoxic lymphocytes

Immmunostimulation
Aim to enhance immunosurveilance and assist the immune system recognise the tumour cell antigens
Principles of immunotherapy
Use a non-specific immunostimulant eg. BCG, SRL-172
Ex-vivo gene therapy can transfect cells with an immunostimulant to activate the immune system
Dendritic cell activation can enhance the body’s recognition of the tumour cell antigens and provide cell mediated killing
The effects of activating the immune system are global and have effects on micrometastasis in addition to the primary
Protects against recurrence as targeting tumour cells using the immune system provides greater specificity and long term protection

9)invasion and metastasis-Inhibitors of HGF/C-Met

E-cadherin helps to assemble epithelial cell sheets and maintain the quiescence of the cells within these sheets
Increased expression of E-cadherin was well established as an antagonist of invasion and metastasis, reduction of its expression was known to potentiate these phenotype
Frequently observed down regulation and occasional mutational inactivation of E-cadherin in human carcinomas provided strong support for its role as a key suppressor of this hallmark capability

10)Tumour promoting Inflammation-selective anti inflammatory drugs

Tumour-associated inflammatory responses promote tumour formation and cancer progression
Inflammatory vasodilation results in increased blood flow, whilst increased permeability of the blood vessels results in exudation (leakage) of plasma proteins and fluid into tissues, which manifests as swelling
Cytokines alter blood vessels to permit migration of leukocytes (mainly neutrophils), to permeate from the blood vessels into the tissue, a process known as extravasation
In addition to cell-derived mediators, several acellular biochemical cascade systems consisting of preformed plasma proteins act in parallel to initiate and propagate the inflammatory response
These include the complement system activated by bacteria, and the coagulation and fibrinolytic systems activated by necrosis and may occur in burns and trauma, as well as cancer
Other bioactive molecules such as growth factors and proangiogenic factors may be released by inflammatory immune cells into the surrounding tumour microenvironment
In particular, the release of reactive oxygen species, which are actively mutagenic will accelerate the genetic evolution of surrounding cancer cells, enhancing growth and contributing to progression of the cancer

61
Q

How can breast cancer present?

A

Mammographic findings: discovered in asymptomatic patients through the use of screening mammography
Breast lump: the most common presenting complaint. The incidence can range from 65-76% of patients
Paget’s disease: associated with intraductal carcinoma involving the terminal ducts of the breast and may have an associated invasive component. Presents as an eczematoid change in the nipple, a breast mass, or bloody nipple discharge
Other local symptoms
Breast pain 5%
Breast enlargement 1%
Skin or nipple retraction 5%
Nipple discharge 2%, nipple crusting or erosion 1%

62
Q

What is the most likely lymph nodes affected and distant metastasis?How does Pagets disease present?

A

ncreasingly, women present as a consequence of mammographic screening. Around 40% of patients will have axillary nodal disease, the likelihood of this rising with increasing size of the primary tumour. The involvement of axillary nodes by tumour is the strongest prognostic predictor. Distant metastases are infrequently present at diagnosis and the commonest sites of spread are: bone (70%), lung (60%), liver (55%), pleura (40%), adrenals (35%), skin (30%) and brain (10–20%).

Paget’s disease of the nipple accounts for 1% of all breast cancer cases and presents with a relatively long history of eczematous change in the nipple area with itching, burning, oozing or bleeding. There may be a palpable underlying lump. The nipple contains malignant cells singularly or in nests. Prognosis is related to the underlying tumour.

63
Q

What are the risk factors for breast cancer?

A

Increasing age is the most important risk factor for most cancers
Family history / personal history due to germline mutation of the BRCA1 and BRCA2 genes and other breast cancer susceptibility genes
Previous benign breast disease, breast tissue density
Reproductive and menstrual history
Early menarche / Late menopause
Nulliparous / late first pregnancy (>35 years)
Oestrogen therapy
OCP?
HRT (relative risk 1.66 in long term users)
Radiation exposure to breast/chest
Obesity (postmenopausal)
Alcohol intake

“Of all women with breast cancer, 5% to 10% may have a germline mutation of the genes BRCA1 and BRCA2. Specific mutations of BRCA1 and BRCA2 are more common in women of Jewish ancestry. The estimated lifetime risk of developing breast cancer for women with BRCA1 and BRCA2 mutations is 40-85%. Carriers with a history of breast cancer have an increased risk of contralateral disease that may be as high as 5% per year. Male BRCA2 mutation carriers also have an increased risk of breast cancer. Mutations in either the BRCA1 or the BRCA2 gene also confer an increased risk of ovarian cancer or other primary cancers. Once a BRCA1 or BRCA2 mutation has been identified, other family members can be referred for genetic counseling and testing.”1

Familial breast cancer

Hereditary predisposition is implicated in around 10% of breast cancer cases
Multiple affected relatives
Young age at diagnosis
Multiple primary cancers
Male breast cancers
Ovarian cancer
Autosomal dominant pattern of inheritance
The overall relative risk of breast cancer in a woman with a positive family history in a first-degree relative (mother, daughter, or sister) is 1.7
Premenopausal onset of the disease in a first-degree relative is associated with a three-fold increase in risk
Postmenopausal diagnosis increases relative risk by only 1.5
If first-degree relative has bilateral disease: 5-fold risk increase
If first-degree relative has bilateral disease prior to menopause: 9-fold risk increase

64
Q

What malignancy will have the greatest risk reduction as a result of COCP use?

A

Ovarian Ca-reduce ovulation

65
Q

What is the most common histological type of breast Ca

A

“Invasive ductal carcinoma

With or without ductal carcinoma in situ is the commonest histology accounting for 70%
Invasive lobular carcinoma

Accounts for most of the remaining cases
Ductal carcinoma in situ (DCIS)

20% of screen-detected breast cancers. It is multifocal in one-third of women and has a high risk of becoming invasive (10% at 5 years following excision only). Pure DCIS does not cause lymph node metastases, although these are found in 2% of cases where nodes are examined, owing to undetected invasive cancer
Lobular carcinoma in situ (LCIS)

A predisposing risk factor for developing cancer in either breast (7% at 10 years)”1

Lymph node progression of involvement

98.7% - demonstrate an orderly progression

54% - level 1 only
23% - levels 1 & 2
21% - levels 1, 2 & 3
1.2% - present at level 2 skipping level 1
0.1% - present at level 3 skipping levels 1 & 2

This allows the use of a sentinel lymph node assessment during surgery. If the first node that would be involved is clear, then no further assessment is required. This saves many patients from the morbidity of a lymph node clearance at the very small risk that some cases may skip a level and give a false negative.

66
Q

When should patients with breast cancer be staged and what Investigations are used?

A

Risk factors for distant metastasis

> 3 involved lymph nodes (p=0.06)
10 involved nodes (p=0.002)
T3/4 tumour (p=0.08)
36/37 patients with metastatic disease had one of these risks
36/269 (13.4%) with risk factors had metastasis
1/807 (0.12%) without risk factors had metastasis

Recommendations for staging

Any patient with:

Tumour > 5 cm
> 3 Nodes (clinically palpable nodes)
Clinical suspicion
All should have CXR, US of liver, bone scan
1 in 7.5 will have metastases
1 in 800 will be missed
CT or MR are used if there is a clinical suspicion of metastasis
Tumour markers? (not as part of staging …)

67
Q

How can germ cell cancers present?

A

Testicular tumours present as a painless solid, unilateral mass in the scrotum, with enlargement of the testicle or swelling, scrotal pain is a feature in 20% of cases, often described as a dragging sensation in the scrotum back pain (10% of cases) can be non-specific or due to retroperitoneal lymph node involvement, or gynaecomastia, seen in 7% of patients (due to high circulating hCG levels).

Features may be similar to epididymo-orchitis, but should raise suspicion if they persist after a course of antibiotics. Quite often it is the girlfriend that finds the tenderness and mass and not the patient. Furthermore, it is often the girlfriend that pesters the patient to go and see the GP.

Ovarian germ cell tumours (GCTs) grow rapidly and the duration of symptoms before presentation is usually 2-4 weeks. There is commonly abdominal pain and a palpable abdominal or pelvic mass causing abdominal distension. Ascites or peritonitis may occur secondary to torsion, infection or rupture. Vaginal bleeding is relatively uncommon. There may be bowel or urinary obstruction, particularly with large masses.

Metastatic disease can manifest as dyspnoea, cough (lung metastasis or PE), SVC obstruction due to a mediastinal mass, ureteric obstruction if retroperitoneal disease is present, CNS symptoms and liver capsular pain. Non-specific features such as weight loss, fatigue, even temperature rise can be seen in advanced disease.

Extra-gonadal tumours may present with chest pain, dyspnoea, SVC obstruction, dysphagia, hoarseness and cough. In the retro-peritoneum, patients experience few symptoms until the tumour is advanced, as it expands without acutely obstructing vital organs. Patients may present with an abdominal mass and abdominal or back pain. Intracranial tumours are extremely rare and present with features of raised intracranial pressure. In more severe cases, neurological symptoms may be overt and be associated with endocrine dysfunction.

You may wish to explore a fertility history as many young patients may not have yet started their family by having children and are likely to have a challenge to their fertility requiring ovum or sperm cryopreservation.

Local trauma is not an aetiological factor for the development of this cancer but it often precipitates self-examination and presentation to a GP.

68
Q

What are the most common sites of metastasis for germ line tumours?

A

Extra-gonadal tumours account for less than 10% of all germ cell tumours. They occur in midline structures, most commonly in the mediastinum (50-70%) and retroperitoneum (30-40%), and rarely the brain, head and neck. In the mediastinum the peak incidence is in the third decade of life.

The crude 5-year overall survival is 95.8% and as men live longer after initial therapy, late toxicities of treatment becomes more important.

69
Q

How can Germ cell cancers be classified?

A

Germ cell tumour-Seminomas/ovarian dysgerminomas(B-HCG),Non seminomas(Embryonal-afp,B-hcg)(Yolk sac tumour-AFP)(Choriocarcinoma-B-HCG)(Immature teratoma-none),Mixed(B-HCG,AFP)

Tumours that are 100% seminoma are considered seminomas. All other tumours, including those that have a mixture of seminoma and non-seminoma components, are considered and should be managed as non-seminomas. Most non-seminomas consist of a mixture of the different germ-cell tumour subtypes. Tumours that appear to have a seminoma histology but are accompanied by an elevated serum level of alpha-fetoprotein (AFP) should be treated as non-seminomas because seminomas do not produce AFP.

Seminoma and dysgerminoma

Seminoma accounts for 50% of malignant germ cell tumours in men, with a peak incidence between the ages of 30-40. Spermatic seminoma is more common in older men, between the ages of 60-70. Ovarian dysgerminoma is directly comparable to testicular seminoma and accounts for the majority of cases of ovarian GCT.

The majority of patients (75%) present with disease confined to the testis or ovary. Spread is usually predictable from para-aortic to supra-diaphragmatic nodes, and then to extra-nodal sites. Growth is slow, and microscopic disease may take up to 10 years to present clinically. Seminomas and dysgerminomas do not produce a reliable tumour marker with which to monitor disease, but hCG can be elevated in 10-25% of cases.

Non-seminomatous germ cell tumours

Disease spread occurs earlier than in seminomas, and therefore only 50% of patients present with localised disease. AFP and hCG tumour markers are elevated in approximately 75% of cases.

70
Q

What are the risk factors for testicular cancer?

A
Family history: 2% report an affected first degree relative, 10-fold increased risk
History of testicular maldescent as a child, orchidopexy (before 2 years of age) only reduces the risk
History of testicular torsion
Kleinfelter syndrome
Down syndrome
Atrophic testis
Previous testicular cancer
Infertility
In utero exposure to oestrogens
71
Q

What investigations are and what are the differentials for a testicular mass?

A

Patients need timely investigation which should include:

Serum tumour markers (AFP, hCG, LDH)

Scrotal ultrasound
Imaging essential for staging is often performed after orchidectomy (CT of chest/abdomen/pelvis)
CT should be within 3 weeks of surgery and should include CT brain if multiple lung metastases and/or serum hCG >10,000

Serum tumour markers should be measured pre-orchidectomy, 24 hours after orchidectomy and weekly thereafter until normal. They are a good marker for residual disease and suggest the absence of residual disease if they normalise with hCG in 24 hours and AFP in 4-6 days.

An ovarian GCT should be suspected in all young women presenting with a pelvic mass, in which case they require ultrasound of the ovaries and often CA-125 is normal.

The differential diagnosis of a testicular mass includes:

Benign epididymal masses are relatively common
Epididymo-orchitis or orchitis (if not resolving within 3 weeks should be referred for urological assessment)
Lymphoma/leukaemic infiltrate

Extra-gonadal tumours may produce placental alkaline phosphatase and gonadal examination and ultrasound for a hidden primary is required, as this alters treatment options and outcomes. CT imaging of the chest, abdomen, thorax (and head if relevant) should be arranged. Genetic analysis may be relevant as trisomy 8 is associated with 16% of cases, as is Kleinfelter syndrome in 14-20%. The tumour should be biopsied and the histological features are the same as for gonadal tumours.

72
Q

What tumour markers can be used in germ cell cancers?

A

“Alpha-fetoprotein (AFP), beta-human chorionic gonadotropin (beta-hCG), and lactase dehydrogenase (LDH) play an important role as serum tumour markers in the staging and monitoring of germ cell tumours and should be measured prior to removing the involved testicle. For patients with non-seminomas, the degree of tumour-marker elevation after the cancerous testicular has been removed is one of the most significant predictors of prognosis. Serum tumour markers are also very useful for monitoring all stages of non-seminomas and for monitoring metastatic seminomas because elevated marker levels are often the earliest sign of relapse.

AFP: Elevation of serum AFP is seen in 40% to 60% of men with non-seminomas. Seminomas do not produce AFP. Men who have an elevated serum AFP are to have a mixed germ cell tumour (i.e., non-seminomatous germ cell tumours [NSGCT]) even if the pathology shows a pure seminoma, unless there is a more persuasive explanation for the elevated AFP, such as liver disease.”1

AFP is physiologically synthesised by the foetal yolk sac, liver and intestine and can be moderately elevated in pancreatic, biliary, gastric and bronchial cancers and non-malignant liver disease. It is grossly elevated in hepatocellular carcinoma and germ cell cancers. It has a longer half life than hCG and therefore there can be a lag after treatment before levels fall and can increase in response to chemotherapy (effect of drugs on the liver)

“Beta-hCG: Elevation of the beta subunit of hCG is found in approximately 14% of the patients with stage I pure seminoma prior to orchiectomy and in about half of patients with metastatic seminoma. Approximately 40% to 60% of men with non-seminomas have an elevated serum beta-hCG.”1 It can be raised in 15% of patients with seminoma and 100% of those with testicular and placental choriocarcinoma. Beta-hCG is a glycoprotein, physiologically formed in the syncytiotrophoblast of the placenta (during pregnancy).

“Significant and unambiguously rising levels of AFP and/or hCG are an indication of relapsed germ cell tumour in most cases and are an indication for treatment even in the absence of radiological evidence of metastatic disease. Nonetheless, tumour-marker elevations do need to be interpreted with caution. For example, false-positive hCG levels can result from cross reactivity of the assay with luteinizing hormone, in which case an intramuscular injection of testosterone should result in normalization of hCG values. There are also clinical reports of marijuana use resulting in elevations of serum hCG and some experts recommend querying patients about drug use and retesting hCG levels after a period of abstinence from marijuana use. Similarly, AFP is chronically mildly elevated in some individuals for unclear reasons and can be substantially elevated by liver disease.

Lactate dehydrogenase (LDH): Seminomas and non-seminomas alike may result in elevated serum LDH but of uncertain prognostic significance as LDH may be elevated in conditions unrelated to cancer. A study of the utility of LDH in 499 patients with testicular germ cell tumour undergoing surveillance after orchiectomy or after treatment of stage II or III disease reported that 7.7% of patient visits had elevations in LDH unrelated to cancer, whereas only 1.4% of visits had cancer-related increases in LDH. Of 15 relapses, LDH was elevated in six and was the first sign of relapse in one. Over 9% of the men had a persistent false-positive increase in LDH. The positive predictive value for an elevated LDH was 12.8%.

A second study reported that among 494 patients with stage I germ cell tumours who subsequently relapsed, 125 had an elevated LDH at the time of relapse. Of these 125, all had other evidence of relapse: 112 had a concurrent rise in AFP and/or hCG, one had CT evidence of relapse before the elevation in LDH, one had palpable disease on examination, and one complained of back pain that led to imaging that revealed retroperitoneal relapse. Measuring LDH appears to have little value during surveillance of germ cell tumours for relapse. On the other hand, for patients with metastatic NSGCT, large studies of prognostic models have found the LDH level to be a significant independent predictor of survival on multivariate analysis.”1

LDH is however and indicator of ‘bulk’ of disease and elevation indicates necrosis as a disease process and therefore raised levels are associated with tumours that grow rapidly.

73
Q

How can germ cell cancers be staged and risk stratified

A

There are two major prognostication models for testicular cancer: staging, and for risk-stratification of men with distant and/or bulky retroperitoneal metastases, the International Germ Cell Cancer Consensus Group classification. The prognosis of testicular germ cell tumours is determined by the following factors:

Histology (seminoma vs. non-seminoma)
The extent to which the tumour has spread (testis only vs. retroperitoneal lymph node involvement vs. pulmonary or distant nodal metastasis vs. non-pulmonary visceral metastasis)
For nonseminomas, the degree to which serum tumour markers are elevated

For men with disseminated seminomas, the main adverse prognostic variable is the presence of metastases to organs other than the lungs (e.g., bone, liver, or brain). For men with disseminated non-seminomas, the following variables are independently associated with poor prognosis:

Metastases to organs other than the lungs.
Highly elevated serum tumour markers.
Tumour that originated in the mediastinum rather than the testis.

Nonetheless, even patients with widespread metastases at presentation, including those with brain metastases, may have curable disease and should be treated with this intent.”

Staging diagram notion

Stage II non-seminoma “is highly curable (>95%). Men with stage II disease and persistently elevated serum tumor markers are generally treated as having stage III disease and receive chemotherapy. For men with normal markers after orchiectomy, non-seminomas are divided into stages IIA, IIB, and IIC for treatment purposes. In general, stage IIA patients undergo retroperitoneal lymph node dissection (RPLND) to confirm the staging. As many as 40% of clinical stage IIA patients will have benign findings at RPLND and will be restaged as having pathological stage I disease. RPLND can thus prevent a significant number of clinical stage IIA patients from receiving unnecessary chemotherapy.

In contrast, stage IIB and IIC patients are usually treated with systemic chemotherapy for disseminated disease because these patients have a higher relapse rate after RPLND. One study reported that by limiting RPLND to patients with earlier stage II disease and normal serum tumour markers, 5-year relapse-free survival (RFS) increased from 78% to 100% after RPLND, while RFS did not change significantly among stage II patients receiving chemotherapy (100% vs. 98%). However, the question of whether to treat patients with stage II non-seminomas germ cell tumours with RPLND or chemotherapy has never been subjected to a randomized trial.”1

74
Q

How can germ cell cancer be treated?

A

“Testicular cancer is broadly divided into seminoma and non-seminoma for treatment planning because seminomatous types of testicular cancer are more sensitive to radiation therapy and chemotherapy and are less prone to distant metastases. Moreover, non-seminomas may include teratomatous elements, which tend to be resistant to chemotherapy and often require surgery for cure. By definition, pure seminomas do not contain elements of teratoma. Therefore, surgery plays a larger role in the management of non-seminomas than in the management of seminomas.

Non-seminomatous testicular tumours include:

Embryonal carcinomas
Yolk sac tumours
Choriocarcinomas
Teratomas
Mixed germ cell tumours"1

For testicular GCT, the management of stage 1 disease is radical inguinal orchiectomy, with clamping of the spermatic cord at the internal inguinal ring to prevent tumour seeding. To prevent seeding of the tumour, pre-operative biopsy is not usually performed. The post-operative tumour markers are monitored during surveillance and if the decline is slow residual disease is possible. The relapse rate after surgery is 20% for seminoma and 30% for non-seminoma tumours. In seminoma, adjuvant radiotherapy to the para-aortic lymph nodes or one cycle of carboplatin can reduce the risk of relapse. In non-seminoma GCT, 2 cycles of BEP chemotherapy can reduce relapse.

In stage 2A and B seminoma, radiotherapy to the para-aortic and iliac lymph nodes is indicated. For all other stages of all forms of GCT, orchidectomy is delayed, as chemotherapy is the mainstay of treatment. In good prognosis disease, 3 cycles of bleomycin, etopiside and cisplatin (BEP) chemotherapy are used, with weekly monitoring of tumour markers, and post treatment CT imaging for response assessment. Orchidectomy is performed after chemotherapy.

Ovarian GCTs are treated with optimal cytoreductive surgery with preservation of fertility considered wherever possible. Total abdominal hysterectomy with bilateral salpingo-oophorectomy (BSO) is therefore avoided in favour of unilateral salpingo-oophorectomy, omentectomy, peritoneal washings and detailed inspection of the abdominal cavity. Biopsies of common sites of spread are taken for staging. Pelvic and para-aortic lymph nodes are biopsied if suspicious. Biopsy of the contralateral ovary is not routine, unless macroscopically abnormal, as potential adhesions or ovarian failure can affect fertility. If it is abnormal, biopsy or ovarian cystectomy is performed followed by bilateral salpingo-oophorectomy if a frozen section demonstrates malignancy or gonadal dysgenesis. Dysgerminoma is the exception, as 10-15% may have bilateral involvement. Patients receive post-operative chemotherapy with 3 cycles of BEP, unless there is bulky residual disease, in which case 4 cycles are given.

Extra-gonadal GCTs are treated dependent upon the site and histological type of tumour. Seminomas are chemotherapy and radiotherapy sensitive, whereas non-seminomas are less so and chemotherapy is given post surgery. For mediastinal and retroperitoneal tumours BEP chemotherapy and surgery are treatments of choice. In intracranial tumours, radiotherapy is given alone in seminoma and in combination with chemotherapy in non-seminoma. Surgery may remove residual mass after chemotherapy but carries a risk of spinal metastases.

75
Q

What is the prognosis of germ cell carcinoma?

A

patients with non-seminoma, good prognosis disease have a 5-year survival of 92-95%, intermediate prognosis tumours have a 70-80% and 48% 5-year survival for poor prognosis patients. Patients with pure seminoma are described as having a good or intermediate prognosis.

76
Q

What are the complications of treatment of germ cell cancer?

A

Because the majority of testis cancer patients who receive adjuvant chemotherapy or radiation therapy are curable, it is necessary to be aware of possible long-term effects of the various treatment modalities, such as the following:

Fertility:All men who require chemotherapy or radiotherapy should be offered sperm storage. “Many patients (50%) have oligospermia or sperm abnormalities before therapy, but semen analysis results generally become more normal after treatment.
“Radiation therapy, used to treat pure seminomatous testicular cancers, can cause fertility problems because of radiation scatter to the remaining testicle during radiation therapy to retroperitoneal lymph nodes

Secondary leukaemia:Several reports of elevated risk of secondary acute leukaemia, primarily non-lymphocytic, have appeared. An increased risk of leukaemia has been associated with platinum-based chemotherapy and radiation therapy. Etoposide-containing regimens are also associated with a risk of secondary acute leukaemia,

Renal function:Minor decreases in creatinine clearance occur (about a 15% decrease, on average) during platinum-based therapy, but these appear to remain stable in the long term and without significant deterioration.

Hearing:Bilateral hearing deficits occur with cisplatin-based chemotherapy, but the deficits generally occur at sound frequencies of 4 kHz to 8 kHz, which is outside the range of conversational tones; therefore, hearing aids are rarely required if standard doses of cisplatin are administered.

Lung function:A study of pulmonary function tests in 1,049 long-term survivors of testis cancer reported a cisplatin-dose-dependent increase in the incidence of restrictive lung disease. Although cisplatin was more strongly associated with decreased lung function in this study, cumulative bleomycin dose was also associated with a decline in forced vital capacity and the 1-second forced expiratory volume (FEV1) but not with restrictive lung disease.

Although acute bleomycin pulmonary toxic effects may occur, they are rarely fatal at total cumulative doses of less than 400 units. Because life-threatening pulmonary toxic effects can occur, the drug should be discontinued if early signs of pulmonary toxic effects develop. Although decreases in pulmonary function are frequent, they are rarely symptomatic and are reversible after the completion of chemotherapy. Survivors of testis cancer who were treated with chemotherapy have been reported to be at increased risk of death from respiratory diseases, but it is unknown whether this finding is related to bleomycin exposure.

Radiation therapy, often used in the management of pure seminomatous germ cell cancers, has been linked to the development of secondary cancers, especially solid tumours in the radiation portal, usually after a latency period of a decade or more. These include melanoma and cancers of the stomach, bladder, colon, rectum, pancreas, lung, pleura, prostate, kidney, connective tissue, and thyroid. Chemotherapy has also been associated with an elevated risk of secondary cancers.”1

Psychosexual issues are common following treatment for testicular cancer and despite good treatment outcomes there remains significant (often unrecognised) psychological morbidity. This can create real issues for relationships particularly in the absence of a long-term partner. Patients can have problems with sexual dysfunction, body image, gender issues and there is a much higher incidence of anxiety and depression.

77
Q

What is Tumour lysis syndrome and how is it treated?

A

This is a metabolic emergency that results from the administration of chemotherapy (usually the first cycle) to a patient with a large volume of cancer, and where the likelihood of response to treatment is high. It can result in hyperuricaemia, hyperkalaemia, hyperphosphataemia, hypocalcaemia and results from the death of large number of tumour cells over a short time period. It can also result in the sudden release of pre-formed hormones that may cause a secondary metabolic emergency.

Adverse consequences of tumour lysis syndrome

Cardiac arrest, arrhythmias
 hypocalcaemia, hyperkalaemia, hyperphosphataemia
Acute renal failure
urate nephropathy
hyperuricaemia
Disseminated intravascular coagulation
cell death
activation of coagulation cascades
intravascular haemolysis (high LDH)

Prevention of tumour lysis

Adequate hydration and urine output (alkalinisation)
Allopurinol or rasburicase as inhibitors of urate oxidase
Beware of drug interactions
ACE inhibitors, spironolactone, NSAIDs
Beware diet
Bananas, chocolate etc.

Predisposing factors for tumour lysis syndrome

Large volume, chemo-sensitive tumour
Burkitt’s lymphoma, other non-Hodgkin lymphoma, leukaemia, small cell lung cancer, germ cell tumours, neuroblastoma, sarcomas
High serum LDH is a clue that the patient is at risk
Renal impairment
Lymphomatous involvement of the kidney
Male, <25 years

Management of tumour lysis syndrome

Requires adequate hydration and strict management of electrolytes but using a low K+ diet, oral and rectal resonium, IV calcium gluconate, IV bicarbonate, insulin/glucose, furosemide, forced hydration and in some cases haemodialysis.

78
Q

What is the significance of detecting gynaecomastia in a patient with a testicular mass?

A

(due to high circulating hCG levels).

79
Q

Should you consider a biopsy of the testicular mass first?

A

markers are monitored during surveillance and if the decline is slow residual disease is possible. The relapse rate after surgery is 20% for seminoma and 30% for non-seminoma tumours. In seminoma, adjuvant radiotherapy to the para-aortic lymph nodes or one cycle of carboplatin can reduce the risk of relapse. In non-seminoma GCT, 2 cycles of BEP chemotherapy can reduce relapse.

80
Q

What is the significance of the raised serum lactate dehydrogenase?

A

LDH is however and indicator of ‘bulk’ of disease and elevation indicates necrosis as a disease process and therefore raised levels are associated with tumours that grow rapidly.

81
Q

What are the clinical presentations of cancer and how is it spread?

A
Direct effects
Mass – pressure, pain, often palpable
Obstruction of a conduit
Ulceration of serosal or mucosal surface, perforation
Metastatic effects
Circulatory invasion
Organomegaly
Metastasis
Effusions
Asymptomatic and detected on investigation
Non-metastatic effects
Temperature
Weight loss
Paraneoplastic effects
Spread
Local
Distant
Blood
Lymphatics
Transcoelomic (intraperitoneal)
82
Q

What Paraneoplastic syndromes can cancer present with?

A
SIADH
Ectopic ACTH
Hypercalcaemia
HPOA
Skin manifestations
83
Q

What are the complications of cancer treatment?

A
Chemotherapy
Neutropenic sepsis
Anaemia (bone marrow suppression)
Venous thromboembolism
Neuropathy
Radiation
Pneumonitis
Toxicity to normal tissue e.g. spinal cord
Surgery
Reduced exercise tolerance
84
Q

In a patient presenting with a Sister Mary Joseph nodule at the umbilicus, what is the most likely method of spread to involve this site?

A

Transcoelomic

85
Q

What are the most typical presenting features of a patient with ovarian cancer and how might that contrast with a patient with Fallopian tube cancer instead?

A

In the early stages, ovarian cancer is a totally insidious disease, producing essentially no symptoms. Tumours may grow to a size of about 10-12 cm before impinging on adjacent organs and producing symptoms of urinary frequency and rectal pressure. Early ovarian cancers may be detected as a pelvic mass noted on a routine pelvic examination.

Advanced-stage disease

Even when advanced, ovarian cancers produce few symptoms
Patients may complain of abdominal bloating or swelling if ascites is present, and large pelvic masses may produce bladder or rectal symptoms
Infrequently, there may be a history of abnormal PV bleeding but this is more common in Fallopian tube cancer
Advanced disease: ascites, indigestion, back ache, complex pelvic masses and omental tumour cake may be present, nodules can frequently be palpated in the pelvic cul-de-sac on rectovaginal examination
Some patients with advanced ovarian cancer have essentially normal-sized ovaries
Backache can be a feature if there is involvement of the para-aortic lymph nodes
It is important to consider and exclude ovarian cancer in women presenting with recent change in bowel habit or vague abdominal symptoms, such as those described in irritable bowel syndrome

Other features:

Constitutional: fatigue, anorexia
Bowel: abdominal bloating or distension, loss of appetite, nausea, vomiting, altered bowel habit, esp. constipation, abdominal pain, bowel obstruction
Kidney: hydronephrosis secondary to ureteric obstruction, haematuria, recurrent UTI, loin pain, renal failure
Pleural effusion: breathlessness, respiratory distress (rare) as a result of a large pleural effusion, which is more common on the right
Thromboembolic phenomenon are seen in advanced cases, particularly clear cell cancer of the ovary
Occasionally, umbilical peritoneal deposits are seen as Sister Mary Joseph nodules indicating transcoelomic spread and stage 4 disease.

86
Q

What are the risk factors for ovarian cancer?

A

Suppressed ovulation appears to protect against the development of ovarian cancer, so pregnancy, prolonged breast feeding and the high-oestrogen contraceptive pill have all been shown to reduce the risk of ovarian cancer.

Up to 7% of women with ovarian cancer have a positive family history. Patients with Peutz-Jeghers syndrome have a 10% risk of ovarian cancer.

Hormonal and reproductive factors that increase risk of ovarian cancer

Low parity and infertility
Ovulation-inducing drugs
Hormone replacement therapy – no association? (controversial)
Oral contraceptives - several large case-control studies have documented a marked protective effect of oral contraceptives against ovarian cancer – particular when take for more than 10 years

Other factors that influence risk of ovarian cancer

Talc - Exposure to talc (hydrous magnesium trisilicate) used as dusting powder on diaphragms and sanitary napkins has been reported to increase the risk of ovarian cancer
Radiation - Data on the association between exposure to ionizing radiation and the risk of ovarian cancer are conflicting
Viruses - Several studies have examined the effect of viral agents, including mumps, rubella, and influenza viruses, on the risk of ovarian cancer. No clear relationship has been demonstrated

Diet

Fat - Countries with a higher per capita consumption of animal fat tend to have higher rates of ovarian cancer
Lactose - Populations with a high dietary intake of lactose who lack the enzyme galactose-1-phosphate uridyltransferase have been reported to be at increased risk
Coffee - Conflicting reports have been published regarding the role of coffee consumption and the risk of ovarian cancer

Risk factors and relative risk of ovarian cancer

                                         Relative risk

General population 1

Family history 17-50

Nulliparity 4-5

1-2 children 3

Use of talc 3

Obesity 2

Oral contraceptive 0.5

87
Q

What familial patterns are seen in ovarian cancer?

A

Two well-recognised familial patterns occur:

hereditary breast/ovarian cancer families, which have mutations in the BRCA1 or BRCA2 gene
Lynch type II families, which have an increased risk of ovarian, endometrial, colorectal and gastric tumours and carry mutations in mismatch repair enzymes

BRCA mutations are inherited as an autosomal-dominant trait with variable penetrance
BRCA1 gene is classified as a tumour suppressor and it appears to play a role in the repair of oxidative damage to DNA. Part of the protein appears to contain a DNA-binding domain, suggesting that it also functions as a transcriptional regulator
The frequency of BRCA1 mutations in the general population is 1 in 800 but Jewish women of eastern European descent: 1 in 100
Women with a germ line mutation of BRCA1 have a significantly elevated risk of both breast and ovarian cancers compared to the general population, at about 90% for breast cancer and 65% for ovarian cancer
BRCA1 mutations are not a feature of sporadic ovarian cancer
Evidence suggests that BRCA1-related ovarian cancers may have a less aggressive clinical course than do sporadic ovarian cancers
HNPCC or BRCA2 mutations - Hereditary ovarian cancers not related to BRCA1 may be related to the HNPCC genes or to the BRCA2 gene

88
Q

What is PARP1 and how can thry be used for treatment?

A

PARP1 is a protein that is important for repairing single-strand breaks (‘nicks’ in the DNA)
If such nicks persist unrepaired until DNA is replicated (which must precede cell division), then the replication itself can cause double-strand breaks to form
Drugs that inhibit PARP1 cause multiple double-strand breaks to form, and in tumours with BRCA1, BRCA2 or PALB2 mutations these double-strand breaks cannot be efficiently repaired, leading to the death of the cells
Normal cells that do not replicate their DNA as often as cancer cells and that lacks any mutated BRCA1 or BRCA2 still have homologous repair operating, which allows them to survive the inhibition of PARP
PARP inhibitors are now in routine use in the clinic
Olaparib, Rucaparib, Niraparib
Following first-line chemotherapy in BRCA mutation carriers (somatic and germ line)
Following second- and third-line chemotherapy
? Role in prevention

he medical oncologist discussed the use of a PARP inhibitor in the form of Olaparib. She had been found to have a germ line mutation in BRCA1 and this treatment can significantly improve the outlook for such patients. This is taken twice daily for 2 years and has to start within 8 weeks of the completion of chemotherapy.

89
Q

How can Ovarian cancer be Investigated?

A

The aim of investigation is to determine the origin of the cancer, determine the extent of spread (staging) and plan appropriate treatment. This patient will require:

Complete medical history and examination
Full blood count
Serum biochemistry
Liver function
Bone profile
Tumour markers: CA-125, CEA, and in younger women: hCG, AFP, LDH (in view of risk of germ cell malignancy)

The combination of transvaginal ultrasound findings, serum CA125 and age can be used to differentiate between benign ovarian cysts and ovarian malignancy, with 80–90% sensitivity and specificity. These investigations have been studied for population screening but have failed to demonstrate a survival advantage from earlier detection.

The mainstay of initial treatment is surgery to remove the cancer and to determine the extent of spread. Therefore for ovarian cancer, staging requires a laparotomy to determine the extent of involvement (not a CT scan).

Exploratory laparotomy: The diagnosis of ovarian cancer is generally made by histopathological study following exploratory laparotomy
Aim of surgery is to debulk the tumour volume (suboptimal >1cm, optimal <1cm, complete)
The stage of the disease can only be determined by surgery
Abdominal ultrasound often the first imaging
Abdominal CT and MRI: are not helpful in making a diagnosis, but may be useful in providing a preoperative assessment of disease extent in probable advanced-stage cases
Preoperative endometrial sampling: Women with abnormal vaginal bleeding should have preoperative endometrial sampling
Preoperative cytological or histological evaluation of effusions or tumour masses can be considered
Paracentesis or needle biopsy only delay definitive management and may lead to seeding of tumour cells along needle tracts

90
Q

What is the risk malignancy index and how is it used in ovarian cancer?

A

Risk of malignancy Index (RMI):

Used to determine the risk of cancer being present in order to stratify who should proceed with the operation. If the risk is low, a local gynaecologist will operate first. If high, the patient should be referred to a surgical centre to see a specialty gynaecological oncologist

RMI = U x M x serum CA-125 value

Requires serum CA-125
Menopausal status (M) scores 1 if premenopausal and 3 if postmenopausal
Ultrasound features (U) = up to max score of 3 (see below)
Ultrasound scans are scored 1 for each of the following:
Multilocular
Solid areas
Bilateral
Ascites
Metastases

RMI Risk % patients Cancer risk %

< 25 low 40 < 3

25-250 moderate 30 20

> 250 high 30 75

An RMI > 200 needs referral to a Gynae Oncologist at a relevant centre

91
Q

How can Ovarian cancer be staged?

A

Staging – simplified version

Stage I Tumour confined to one ovary

Stage II Tumour extension to nearby organs

Stage III Tumour involvement throughout the abdominal cavity

Stage IV Widespread disease, parenchymal liver metastasis, outside the abdomen

5-year survival by stage at presentation

FIGO stage Proportion of cases (%) 5-year survival (%)

Ia 19.3 92.1

Ib 2.7 84.9

Ic 8.1 82.4

IIa 2.7 69.0

IIb 4.2 56.4

IIc 3.0 51.4

IIIa 6.9 39.3

IIIb 6.6 25.5

IIIc 18.0 17.1

IV 28.3 11.6

92
Q

How can Ovarian cancer be treated?

A

Surgery remains the first intervention for patients with ovarian cancer, with the intention of best effort debulking of the disease to achieve complete (no macroscopic disease), optimal (macroscopic disease <1cm) or suboptimal debulking (residual disease >1cm). Surgery involves a laparotomy, total hysterectomy, bilateral salpingo-oopherectomy with omentectomy and lymph node resection.

Following surgery the majority of women will be candidates for adjuvant chemotherapy with carboplatin and paclitaxel but the addition of bevacizumab for those with high-risk disease is advantageous. Neoadjuvant chemotherapy is used for patients with extensive disease at presentation, for whom surgery may not be initially possible (due to fitness, complications such as PE, or operability) with the aim of shrinking the disease in order to consider interval debulking. Non-epithelial ovarian cancer requires surgery followed by chemotherapy, based upon the predominant cell type present. From diagnosis 70-80% of patients achieve remission, 15% have residual disease at the end of chemotherapy and 5-7% progress on initial chemotherapy.

At relapse, second-line chemotherapy is associated with a response rate of 20–40%, with higher rates correlating with greater treatment-free intervals. Serum CA125 may be useful in predicting relapse (median 4.2 months ahead), and predicting response to treatment but early treatment based upon the tumour marker alone does not give a survival advantage. Approximately 60% of patients with ovarian cancer will relapse at some point. Hormonal approaches using tamoxifen or aromatase inhibitors can slow down the rate of progression and delay onset of symptoms.

93
Q

What are the disease related complications for Ovarian cancer?

A

Not all patient respond completely to their adjuvant treatment and many (60%) will eventually relapse. Advanced disease will spread in a transcoelomic manner producing ascites and can produce sub-acute or complete bowel obstruction due to serosal involvement of the bowel. A pelvic mass can result in hydronephrosis due to ureteric obstruction. Ascites may require frequent paracentesis and talc pleurodesis can reduce the recurrence of pleural effusions. Patients with ovarian cancer are at a particularly high risk of thrombosis (DVT, PE) due to a prothrombotic tendency that correlates with the disease activity. Furthermore, thrombosis can occur despite adequate anticoagulation.

Local invasion
Lymphoedema
Vaginal discharge
Bowel obstruction
Ascites, pleural effusion
Distant metastasis
Liver, lung, bone, brain
Non-metastatic
Pulmonary emboli
Dermatomyositis
94
Q

How can ascites be managed in a patient with cancer?

A

Management of ascites (in any patient with cancer):

The collection of intraperitoneal fluid in a patient with known malignancy is most likely due to involvement of the peritoneum with serosal implants
Ascites at initial diagnosis may have prognostic significance and should be investigated to demonstrate the presence of malignant cells on cytological assessment
Ascites should not influence the appropriate investigation of patients as curative intent may still be possible depending on the primary site of the cancer
Diagnosis is usually on clinical grounds but ultrasound, MRI or CT imaging are useful for guided drainage, particularly where the fluid is pocketed
Non-malignant causes of ascites include congestive heart failure, liver cirrhosis, renal failure, hypoproteinemia, infectious processes, and endometriosis
Approximately 10% of patients with ascites have an underlying malignancy, although in a patient with advanced cancer it is the most likely diagnosis
Investigation for cytology is indicated when a definitive diagnosis of malignant ascites is necessary for staging purposes or when planning surgical intervention
Patients with malignant ascites secondary to ovarian cancer have a significantly better outcome and therefore, thorough investigation of female patients presenting with ascites of unknown origin should be undertaken
Most common treatment is paracentesis, including the use of indwelling (Pleurx) catheters
Loop diuretics, salt restriction, and aldosterone-inhibiting diuretics are generally not beneficial since sodium retention is not a cause of malignant ascites
The use of albumin has never proven beneficial in delaying fluid reaccumulation nor been more effective than crystalloid solutions in restoring intravascular volume depletion after drainage of large quantities of peritoneal fluid

95
Q

What is the prognosis of Ovarian cancer?

A

Prognosis correlates with stage at diagnosis. Overall 5-year survival is 30% but ranges from >90% for stage 1 to <25% for stage 4. Patients that have disease resistant to platinum therapy, large volume residual disease following debulking, or clear cell histology all have a worse outcome. Patients with BRCA gene mutations are more likely to have visceral metastasis, but equally are more likely to respond to platinum therapy and have longer treatment-free intervals. The best predictor of outcome is for patients that achieve complete cytoreduction at initial surgery.

96
Q

What is the most likely cause of death due to cancer in women

A

Lung Ca

97
Q

What Factors that must be considered when assessing any screening programme?

A

Factors that must be considered when assessing any screening programme:
Is the disease curable if diagnosed early?
What is the sensitivity of the test used?
Is the disease common?
How frequently should the test be done?
What population should be tested?
What are the disadvantages of screening?

SPECIFICITY (Ability to detect NEGATIVES)
= test negatives/(test negatives + false positives)

SENSITIVITY (Ability to detect POSITIVES)
= test positives/(test positives + false negatives)

POSITIVE PREDICTIVE VALUE
= test positives/(test positives + false positives)

NEGATIVE PREDICTIVE VALUE
= test negatives/(test negatives + false negatives)

98
Q

How is cancer screening programme evaluated?

A
Evaluation should include:
Screening uptake rate in population
Recall rate of screened population (true positives + false positives)
Biopsy rate
Cancer detection rate
Rate of interval cancers (cancers between screening tests)
Incidence rate in non-attendees
Deaths from cancers

Sources of bias in evaluation
Lead time bias
By detecting tumours at an earlier (presymptomatic) stage, the subsequent survival is spuriously prolonged when compared with a symptomatic cohort

Length time bias
At the start of a screening programme, more patients with indolent disease will be detected initially (since at any one time the prevalence of slowly-growing tumours will be greater, even if the incidence of aggressive tumours is similar)
This bias leads to an illusory survival improvement in the screened cohort

99
Q

How is screening done for common cancers?

A

Gastric cancer

Early stage gastric cancer is surgically curable
In Japan where the incidence of gastric cancer is high, endoscopic screening has increased the number of cancers that are detected at this early stage and are cured

Prostate cancer

An early screening trial of serum PSA measurement and digital rectal examination in 18,000 men detected a cancer rate of 3.5% and >90% were localised tumours who were candidates for radical curative therapy
This lead to enthusiasm for prostate cancer screening particularly in the US
However, no randomised clinical trials (RCT) have adequately addressed the impact of screening on survival
no RCT have evaluated the optimal therapy for localised early prostate cancer

Models of screening in the US reveal part of the reason for this: 3% men are expected to die of prostate cancer and the average life reduction is 9 years
For 100 men an ideal screening programme coupled to a complete curative therapy could prevent 3 deaths and gain 27 years of life
This translates to an increased life expectancy for the whole screened cohort of 3 months

Cervical cancer

Exfolliative cytology and Papanicolaou staining form the basis of cervical smear screening for the detection of pre-malignant cervical intraepithelial neoplasia (CIN)
The cervical screening programme has reduced the incidence of squamous cell carcinoma of the cervix but is not able to detect adenocarcinoma which frequently develop deeper in the cervix and account for 15% of invasive cervical cancer
Abnormal smears (CIN 2/3) should be followed by colposcopy (visualisation of cervix under 10-15 power magnification with bright light and green filter to enhance vascular pattern) and biopsy
If colposcopic biopsy is incomplete patients should proceed to cone biopsy removing the transition zone

Ovarian cancer

Ovarian cancer screening
Ovarian screening by serum CA-125 tumour marker measurement and/or transvaginal ultrasound is under investigation in randomised controlled trials
Although this approach is feasible and can detect tumours at an earlier, and in theory, more curable stage, there is as yet no evidence of improved survival in screened cohorts
Even in women with BRCA 1 or 2 there is no evidence to support screening

Lung cancer

Lung cancer screening
Screening for lung cancer by CXR and/or sputum cytology has not been found to be effective, even in high risk populations
Spiral CT can detect early lung cancers and is still in clinical trial
? Management of <1 cm spiculated lesions

100
Q

What are the key cancer staging committees?

A

UICC
European; WHO
AJCC
American
FIGO
Fédération Internationale de Gynécologie et d’Obstétrique
TNM aligned with stage grouping & FIGO stages (clinical)

101
Q

What are aims of cancer staging?

A

1) Aid treatment planning
2) Give some indication of prognosis
3) Assist in evaluation of treatments (reproducibly)
4) Facilitate information exchange
5) Research

102
Q

What are principles of cancer staging?

A

Grouping by survival rates (early vs late)
Classification by anatomical extent (originally clinical)
Now supplemented by investigations
pTNM: pathological classification post-surgery
If doubt, choose less advanced category

103
Q

What is the TNM staging

A
T – primary tumour
Local tumour spread
N – regional lymph nodes
Lymphatic spread
M – distant metastases
Blood-borne spread

TNM is precise
4 ‘T’ categories, 3 ‘N’ and 2 ‘M’  24 TNM combinations
Stage grouping:
aids analysis
aims to group TNM combinations by survival
Stage I & II: localised to organ of origin
Stage III: locally extensive spread (regional LNs)
St
Different frameworks:
Direction or ease of spread
Size of tumour
Combination of above
Non-conformists
age IV: distant metastases

T1-T4
Different frameworks:
Direction or ease of spread
Size of tumour
Combination of above
Non-conformists
N stage
Less predictable
Classified by
Presence
Size
Site
Number
Combinations thereof

M stage

2 categories
M0: curable
M1: invariably fatal
Subcategories
M1a
M1b
(M1c)
e.g. Prostate: non-regional LN, bone, other sites
104
Q

What does a cancer MDT consist?

A

A group of healthcare professionals that work together to provide high quality clinical services to patients, increasing efficiency and producing well rounded care
The core team is made of up of the health professionals directly involved in patients care, such as physicians, surgeons, radiologist, pathologist, specialist nurses
The wider team encompasses professionals that have an adjunctive role, such as occupational therapists, physiotherapists

105
Q

What is the Role of MDT?

A

The MDT is well established in oncology
Cancer is a multisystem disease requiring input from a variety of disciplines
The team meet on a regular basis, usually weekly, to discuss the patient’s progress and provide a forum for interdisciplinary communication allowing for coordination of care and decision making
The meeting is designed to be patient-centred and provides an opportunity for communication between the different disciplines, planning of investigations and management

It is a platform on which individual clinicians can discuss complex cases or situations and draw on the collective experience of the team membership to decide on the best approach for an individual patient
This can be particularly important when discussing the patients with a rare condition or situation

Specific roles

Plan diagnostic and staging investigations
Decide on the appropriate primary treatment modality (most commonly surgery but the use of neoadjuvant chemotherapy before interval surgery is increasing)
Arrange review by the oncologists to plan the assessment of the patient prior to systemic therapy or radiotherapy
Discuss the additional support requirements for the individual patient, such as; physiotherapy, psychological support, symptom control, nutritional care or rehabilitation in the post-operative period

Agree operational policies to deliver high quality care to patients, possibly at a network level
Plan surveillance strategies for individual patients
Ensure the appropriate transition from treatment with curative intent to that of palliation of symptoms
Promote recruitment into clinical trials
Audit the MDT data to ensure the delivery of quality care to patients by the team

106
Q

What are the benefits of MDT?

A

Studies have shown that multidisciplinary teams can improve the quality of life, lower mortality, and reduce the cost of cancer care
Early involvement of the MDT can facilitate appropriate and timely treatment, which is more likely evidence-based and can lead to greater patient satisfaction
The MDT working well together can benefit the healthcare professionals, with studies showing greater job satisfaction of those involved

Allow inter-speciality working providing a basis for support and education for the whole team
Improved coordination between services, reducing duplication of investigations and minimising inefficient communication
More efficient, saving time, money and resources, which inevitably lead to a greater standard of care for the patient, and wider benefit to the NHS

107
Q

What are combined clinics?

A

A number of MDTs run a combined clinic in which several team members are present
Patients can be seen by more than one specialist in the same hospital visit, providing a one-stop opportunity to review and plan treatment
This approach negates the requirement for writing referral letters, improves communication between team members, reduces delays as patients can be reviewed by more than one specialist and is more efficient for the patient
Joint decisions relating to treatment can be decided and implemented in the minimal amount of time, thereby shorting the time to initiation of treatment

108
Q

What are the red flags signs and symptoms of cancer?

A

Persistent cough or hoarseness – could indicate lung cancer

A change in the appearance of a mole – could mean you’re suffering skin cancer

A persistent change in bowel habits – could be a sign of bowel cancer

A sore that does not heal – depends on where, a mouth ulcer could mean mouth cancer

Persistent difficulty swallowing – can mean a person is suffering oesophageal cancer

Unexplained weight loss – can indicate several types of cancer

Persistent change in bladder habits – could be a sign of bladder cancer and prostate cancer in men

An unexplained lump – can be a warning sign of many forms of the disease

Persistent unexplained pain – depending on where, can denote many types of cancer

Unexplained bleeding – depends where but can mean bowel, cervical or vulval cancer

109
Q

How can corrected calcium be calculated?

A

The most common metabolic complication of malignancy
Oncological emergency
Seen in up to 40% of cancer patients and indicates a poor prognosis
Particularly common in multiple myeloma, breast, lung, kidney, head and neck, and lymphoma
Clinical presentation is often non-specific, and may mimic deterioration due to progressive disease

Serum calcium values should be corrected

corrected Ca++ = serum Ca++ + [40-serum albumin (g/L)x0.02]

110
Q

What are the causes of confusion in cancer?

A

In any patient with cancer that presents with confusion consider:

Hypercalcaemia
Brain metastasis
Metabolic disturbance

111
Q

What is hypercalcemia in malignancy and how is it treated?

A

A common problem but in many cases is due to bony metastases
True paraneoplastic hypercalcaemia is due to tumour production of parathyroid hormone-related protein
PTH-rP produces hypercalcaemia of malignancy without bone metastasis
It is 100-fold more potent than PTH
10% of patients with cancer develop hypercalcaemia

Cancer types

NSCLC, Head and neck, Renal, SCC of oesophagus
Note: is rare in breast cancer where hypercalcaemia is usually due to bone metastases

Clinical presentation

Rapid onset nausea, polyuria, polydipsia, dehydration, cardiac arrhythmias

Diagnosis

Serum Ca2+ > 2.7 mmol/l, serum chloride low, hypercalcuria, high urinary phosphate, low or undetectable plasma parathyroid hormone

Treatment

Saline hydration, IV bisphosphonate

112
Q

What is the liklihood of a primary be indentified in cancer of unknown primary?

A
  • Even with extensive investigation, less than 20% of patients have a primary site identified antemortem
  • Even at autopsy, 70% of cases remain undiagnosed
  • Primary sites are most frequently detected in the lung and pancreas, followed by other gastrointestinal and gynaecological malignancies
113
Q

What are the prognostic factors for cancer of unkown primary?

A
Limited life expectancy with median survival of 6-9 months
Poorly differentiated carcinoma, squamous cell carcinoma, neuroendocrine carcinoma
Lymph node involvement (except supraclavicular fossa)
Number of metastatic sites
Female sex (males do worse)
Performance status (less fit do worse)
Weight loss (>10% do worse)
Serum markers (alk phos, LDH, CEA)
114
Q

What are common causes of transudate?

A

Common causes of a transudate:

‘Failure’
Heart failure
Liver failure
Kidney failure
Thyroid failure
Respiratory failure (rare)
Meig’s syndrome
115
Q

What can cause a raised Ca 125?

A

Can be elevated in any cause of

Inflammation
Infection
Infarction
And almost any cancer….

Extracellular fluid space can act as a reservoir (pleural effusions and ascites) and therefore, it is never diagnostic.

116
Q

How can cancers of unknown primary be treated?

A

As already outlined in this case, the goal is to identify cancers that are potentially curable or very treatable and for all those that are left, the aim of treatment is palliation of their symptoms.

The Manual of Cancer Service Standards requires that treatment intent is recorded in the medical record as either curative or palliative. In this case palliative intent, palliation means improvement in symptoms and therefore the primary goal must be improvement in quality of life. Any improvement in duration of life is a secondary goal.

Factors that can influence the choice of treatment include:

Age – not in isolation
Fitness (performance status) most important
Co-morbid conditions
Organ impairment (more challenging if there is an organ in failure)
Patient priorities

117
Q

What Psychological issues need to be addressed?

A

Distress: most patients with a terminal diagnosis undergo some degree of emotional distress. Multiple factors may be responsible for this distress and it is important to attempt to address as many as possible. Good communication skills are vital when discussing prognosis with the patient and family, incorporating both honesty and reassurance where appropriate. Involving the patient and family in decisions about their care and respecting patient autonomy is essential. Control of physical symptoms can be of great benefit to a patient’s mental state.

Depression: is common in end of life patients and it is crucial to distinguish normal psychological distress from significant clinical depression. Physical symptoms of depression such as lack of energy and loss of appetite may be masked by the biological progression of cancer. However, appropriate screening tools such as the Hospital Anxiety and Depression (HAD) scale can help identify patients with depression. Treatment should be holistic, incorporating additional psychological support, symptom control and antidepressant medication if necessary.

Spirituality: the patient’s beliefs, thoughts, hopes and fears about the end of their life should be explored with them to ensure that appropriate spiritual support is given. This includes, but is not limited to, religious faiths and cultural ideas around death and dying.

Bereavement: support for patients and their families coming to terms with the prospect of dying is important. Grief can develop both before and after a death, and informal support, as well as psychological therapy, can be very beneficial. Cancer support groups are extremely helpful in helping patients and relatives understand the disease and deal with its consequences

118
Q

What malginancies can cause ascites?

A

Ovarian,peritoneal,liver,pancreatic,stomach,breast,uterine and cervical

119
Q

If distant disease is present at diagnosis, what is the most common site of distant metastasis in breast cancer?

A

Bone

120
Q

Which lymph node areas must be examined in a patient suspected of having a breast cancer?

A

Breast cancer usually presents as a mass that persists throughout the menstrual cycle. A nipple discharge occurs in 10% and pain in only 7% of patients. Less common presentations include inflammatory carcinoma with diffuse induration of the skin of the breast, and this confers an adverse prognosis. Increasingly, women present as a consequence of mammographic screening. Around 40% of patients will have axillary nodal disease, the likelihood of this rising with increasing size of the primary tumour. The involvement of axillary nodes by tumour is the strongest prognostic predictor. Distant metastases are infrequently present at diagnosis and the commonest sites of spread are: bone (70%), lung (60%), liver (55%), pleura (40%), adrenals (35%), skin (30%) and brain (10–20%).

Paget’s disease of the nipple accounts for 1% of all breast cancer cases and presents with a relatively long history of eczematous change in the nipple area with itching, burning, oozing or bleeding. There may be a palpable underlying lump. The nipple contains malignant cells singularly or in nests. Prognosis is related to the underlying tumour.

Neck and axilla: lymphadenopathy
Nipple: is there discharge or retraction?
Breast tissue: is there discolouration, oedema, peau d’orange, erythema, nodules, ulceration, lack of symmetry, skin thickening?
Chest: signs of consolidation, nodules in skin
Abdomen: hepatomegaly
MSK: focal tenderness in axial and peripheral skeleton due to bone involvement

121
Q

What are the different histological types of breast cancer and what is the commonest?

A

Invasive ductal carcinoma

With or without ductal carcinoma in situ is the commonest histology accounting for 70%
Invasive lobular carcinoma

Accounts for most of the remaining cases
Ductal carcinoma in situ (DCIS)

20% of screen-detected breast cancers. It is multifocal in one-third of women and has a high risk of becoming invasive (10% at 5 years following excision only). Pure DCIS does not cause lymph node metastases, although these are found in 2% of cases where nodes are examined, owing to undetected invasive cancer
Lobular carcinoma in situ (LCIS)

A predisposing risk factor for developing cancer in either breast (7% at 10 years)”1

122
Q

How can a breast mass be evaluated?

A

Evaluation of a cystic mass in the breast

Fine-needle aspiration (FNA) - If the mass is a cyst it can simply be aspirated with a fine needle, which should yield non-bloody fluid and result in complete resolution of the lesion
Ultrasonography is used to determine whether a lesion is solid or cystic, and whether a cyst is simple or complex
Biopsy: a biopsy is indicated if the cyst fluid is bloody, the lesion does not resolve completely after aspiration, or the cyst recurs after repeated aspirations
Note that: cystic carcinoma accounts for < 1% of all breast cancers and an intraluminal solid mass is a concerning sign suggesting (intra) cystic carcinoma, and should be biopsied.

Evaluation of a solid mass in the breast

The decision to observe a patient with a breast mass that appears to be benign should be made only after careful clinical, radiological, and cytological examinations
Mammography can be used to assess radiological characteristics of the mass and evaluation of the remainder of the ipsilateral breast as well as the contralateral breast
Fine needle aspiration is a simple method for obtaining material for cytological examination. False-positive results from 0%-2.5% and false-negatives varies from 3-27%
A core biopsy (18 gauge or larger needle biopsy) can be advantageous since architectural as well as cellular characteristics can be evaluated. An excisional biopsy, in which the entire breast mass is removed, definitively establishes the diagnosis

Evaluation of a non-palpable mass

These are normally found on imaging such as mammography and can be approached by:

Wire excision biopsy
Stereotactic-guided core biopsies
Ultrasound-guided core biopsies
Breast MR imaging for more characterisation

123
Q

What tumour markers can be used in breast cancer?

A

Elevated serum levels of CA 15.3 are found in 12.5% of women with benign breast disease, preoperatively in 11% of women with operable breast cancer, and in 64% of women with metastatic breast cancer
It has no value in screening because of low sensitivity for the early stages of disease
False positive: Elevated in gynaecological cancers
CA 15.3 elevation increases with increasing stage of disease and highest levels are seen in patients with liver or bone metastases
It is not accurate enough to be used alone to define response
Several trials have shown that a rising CA 15.3 level during follow-up can detect relapse 2-9 months before clinical signs or symptoms develop
Rising levels indicated recurrence in 73% of those with a recurrence and in 6% of those without a recurrence
It therefore has no role in diagnosi

124
Q

How can breast cancer be prevented?

A

The main strategies for preventing breast cancer include:

Prophylactic mastectomy which reduces risk of breast cancer because it reduces volume of breast tissue and in patients with BRCA mutations, risk is reduced to 1%
Ovarian ablation (makes the patient postmenopausal, thereby reducing exposure to oestrogen)
Drugs such as tamoxifen as a prevention treatment
Increased, more intensive surveillance
It is important to consider that in a patient with unilateral breast cancer, that their treatment may also be preventing a contralateral cancer

Breast cancer prevention studies

Chemoprevention with tamoxifen has been shown to reduce the incidence of breast cancer in an American randomized controlled trial of 13,000 healthy women at high risk of developing breast cancer. However, these results have not been reproduced in two similar European trials.

NSABP P1 trial results

Tamoxifen advantages
49% less invasive cancer (p<0.00001)
50% less DCIS (p<0.002)
Only prevented ER+ tumours
Tamoxifen disadvantages
2.53 x more endometrial cancer
Increased DVT and PE risk
No clear cut impact on SURVIVAL

Aromatase inhibitors in prevention

Secondary prevention has been an endpoint in adjuvant studies of aromatase inhibitors

Aromatase inhibitors reduce contralateral cancer incidence
Now being used in prevention trials in postmenopausal women

125
Q

What are the complications of breast cancer?

A

Breast can produce a number of complications that can be local or distant:

Local invasion
Lymphoedema
Pleural effusion, ascites
Distant metastasis
Bone, liver, lung, brain
Spinal cord compression
Non-metastatic
Hypercalcaemia
126
Q

What cancers can cause bony metastasis and how can thry be treated?

A

Most of the people who die of cancer each year have tumour metastasis
Bone is the third most common organ involved by metastasis, behind lung and liver
In breast cancer, bone is the second most common site of metastatic spread, and 90% of patients dying of breast cancer have bone metastasis
Breast and prostate cancers metastasise to bone most frequently, which reflects the high incidence of both of these tumours, as well as their prolonged clinical courses
Other tumours that commonly cause symptomatic bone metastases include kidney and thyroid cancer, and multiple myeloma
Patients with bone metastasis from breast cancer have an average 2-year survival from the time of presentation with their first bone lesion
More patients are living with bone metastases, and thus the challenge is to improve their quality of life
Early detection and aggressive management of metastases is the goal
Maintain and maximize patients’ quality of life and functional level
Currently, care is optimised in only a fraction of patients with bone metastases

There are four main goals in managing patients with metastatic disease to the skeleton:

pain relief
preservation and restoration of function
skeletal stabilization
local tumour control (e.g., relief of tumour impingement on normal structures, prevention of release of chemical mediators that have local and systemic effects)

127
Q

What cancers can cause spinal cord compresion and how does it present?

A

Malignant spinal cord compression is defined as the compressive indentation, displacement, or encasement of the spinal cord’s thecal sac by metastatic or locally advanced cancer. The finding of bilateral UMN signs should be considered spinal cord compression until proved otherwise. Spinal cord compression from metastatic cancer remains an important source of morbidity despite the fact that with early diagnosis, treatment is effective in 90% of patients. Any cancer capable of metastasis or local invasion can produce malignant spinal cord compression.

The diagnosis is made clinically and confirmed radiologically. MR is investigation of choice, and the incidence of involvement location is:

Thoracic 70%
Lumbar 15%
Cervical 10%
Sacral 5%

Response to non-surgical therapy and the duration of survival following treatment can vary considerably among different histological tumour types and the degree of pre-treatment neurological dysfunction is the strongest predictor of treatment outcome. Ambulation can be preserved in greater than 80% of patients who are ambulatory at presentation and the key to successful management is a heightened awareness of signs and symptoms, specifically newly developed back pain or motor dysfunction, leading to early diagnosis and treatment.

Underlying cause of spinal cord compression

Tumour type

Frequency %

Breast cancer

29
Lung cancer

17
Prostate

14
Myeloma

4
Renal

4
Lymphoma

5
Sarcoma

2
Other

23

128
Q

What is the earliest clinical feature of spinal cord compression?

A

Vertebral pain, especially on coughing or sneezing

129
Q

What are the endocrine manifestations of cancer(paraneoplastic syndrome)

A
The range of syndromes include:
Endocrinologic
Haematologic
Gastrointestinal (GI)
Renal
Cutaneous
Neurologic

In 1928, syndrome of ectopic ACTH overproduction was described in a patient with SCLC
Produced hirsutism, diabetes mellitus, hypertension, and adrenal hyperplasia
Tumours produce ACTH or an ACTH-like substance, which leads to adrenal hyperplasia and hypercortisolism, coining the term ectopic ACTH production
Subsequently, propiomelanocortin (POMC), the precursor hormone of ACTH, was described, containing not only ACTH, but melanocyte stimulating hormone, beta-lipotropin, endorphins, and enkephalins

130
Q

How does ectopic ACTH production present and what are causes?

A
Inappropriate overproduction of adenocorticotrophic hormone (ACTH) precursors: ACTH and MSH, b-lipotropin, endorphins, encephalins
Cushing’s syndrome
truncal obesity
hypertension
fatigue
depression
weakness
hirsutism
hypokalaemia, metabolic alkalosis
hyperpigmentation
oedema

Cancer types
SCLC (50% of cases of ectopic ACTH, 7% of SCLC), NSCLC, pancreatic, thymic and carcinoid tumours, phaeochromocytoma, medullary carcinoma of the thyroid
Presentation
rapid onset, marked weakness secondary to proximal myopathy, hyper-pigmentation, metabolic disturbances e.g. hyperglycaemia, hypokalaemic alkalosis
Diagnosis
Clinical features, esp. hyperpigmentation, myopathy; hypokalaemia and metabolic alkalosis; high 24hr urinary cortisol, high plasma ACTH/precursors, no response to high-dose dexamethasone suppression or corticotropin-releasing hormone stimulation
Treatment
Specific anti-tumour treatment. Decrease cortisol secretion either surgically (bilateral adrenalectomy) or medically (octreotide, ketoconazole, aminogluthethamide

131
Q

What are the causes of said?how does it present?management?

A

The most common endocrine paraneoplastic syndrome
Due to inappropriate secretion of anti-diuretic hormone (arginine-vasopressin)

Causes of SIAD
CNS
Infections, vasculitis, stroke, head injury, tumours, Guillain-Barre syndrome, acute intermittent porphyria, psychological stress
Pulmonary
Infections, tumours, positive pressure ventilation, pneumothorax, asthma, cystic fibrosis
Drugs
Vincristine, Cyclophosphamide, Morphine, Chlorpropramide, Thiazides, Clofibrate, Carbamazepine, Cisplatin

Cancer types
SCLC (10% of patients), pancreatic, prostate, NHL, HD

Presentation
Often asymptomatic; CNS toxicity - fatigue, headaches; progressing to altered mental state, confusion and seizures

Diagnosis
exclude non-malignant causes e.g. CNS disease (infection, trauma, vascular) and pulmonary disease (infections, cystic lesions, asthma), drug-induced (thiazides, cytotoxics, narcotics); clinically euvolemic; laboratory studies (see below)

Treatment
Fluid restriction (0.5-1.0 L/day); democlocycline
132
Q

What is the diagnostic criteria for SIADH?

A
Hyponatremia Na+ <130 mmol/L
Normal serum albumin and glucose
Serum hypo-osmolarity <275 mosm/Kg
Urine osmolarity > serum osmolarity
Urinary sodium >25mmol/L
Non-suppressed ADH
133
Q

What type of tumours secrete gonadotrophin?

A

Secretion may occur in patients with pituitary tumours, gestational trophoblastic tumours, germ cell tumours, hepatoblastomas in children and lung tumours
Frequency of signs associated with raised b-hCG – most commonly seen is gynaecomastia in men

Investigations
Testicular exam
CXR / CT scans

Other extragonadal tumours that secrete b-hCG
Lung
Adrenal
Hepatoma
Gastrointestinal tract tumours
Genitourinary tumours
134
Q

What cancers can cause hypercalcemia and how?

A

A common problem but in many cases is due to bony metastases
True paraneoplastic hypercalcaemia is due to tumour production of parathyroid hormone-related protein
PTH-rP produces hypercalcaemia of malignancy without bone metastasis
It is 100-fold more potent than PTH
10% of patients with cancer develop hypercalcaemia

Cancer types
SCC-NSCLC, Head and neck, Renal, SCC of oesophagus
Note: is rare in breast cancer where hypercalcaemia is usually due to bone metastases
Presentation
Rapid onset nausea, polyuria, polydipsia, dehydration, cardiac arrhythmias
Diagnosis
Serum Ca2+ > 2.7 mmol/l, serum chloride low, hypercalcuria, high urinary phosphate, low or undetectable plasma parathyroid hormone
Treatment
Saline hydration, IV bisphosphonate

135
Q

What can cause Encephalomyopathies and how does it present?

A

Perivascular inflammation and selective neuronal degeneration at several levels of the nervous system
Can affect the limbic system, brainstem and spinal cord. Loss of neurons in the amygdala, hippocampus and insular cortex gliosis, lymphocyte cuffing of blood vessels, microglial nodules
Cancer types
SCLC (75% of cases), breast, ovary, NHL
Presentation
Slow, subacute onset; progressive, loss of short term memory, hallucinations, fits, personality changes
Diagnosis
CSF: raised protein/IgG level, pleocytosis; Serum: anti-Hu antibody; MRI
Treatment
Anti-tumour therapy

136
Q

What is Lambert Eaton myasthenic syndrome?How does it present?

A

Disorder of the neuromuscular junction; reduced pre-synaptic calcium-dependent acetylcholine release
About 60% of patients with LEMS have underlying cancer
Cancer types
SCLC (60-70%), breast, thymus, GIT cancers
Presentation
Proximal muscle weakness, Classical myaesthenia gravis affects bulbar muscles; LEMS does not, yet 30% will have dysphagia
Diagnosis
EMG: normal conduction velocity with low amplitude compound muscle action potential that enhance to near normal following exercise
Treatment
Cancer treatment, corticosteroids, plasma exchange (high response rate), cholinesterase inhibitors usually ineffective

137
Q

What is Dermatomyositis/polymositis and does it present?

A

Inflammatory myopathies, often present prior to cancer diagnosis
Cancer types
NSCLC, SCLC, breast, ovary, GIT cancers, associated with cancer in only 10% of cases
Presentation
proximal myopathy, skin changes, other systemic features: cardiopulmonary conditions, arthralgias, retinopathy
Diagnosis
Serum: high CK, LDH, aldolase; muscle biopsy: myositis and inflammatory degeneration; EMG: fibrillation, insertion irritability, short polyphasic motor units
Treatment
Search for and treat tumour; corticosteroids, azathioprine, inconsistent course, independent of tumour

138
Q

What are the Haematological manifestation of paraneoplastic syndrome?

A

Paraneoplastic syndromes that involve haematopoietic cells and clotting factors are extremely common
Often related to hormones and growth factors that regulate haematopoiesis

Erythrocytosis
Anaemia
Granulocytosis/granulocytopenia
Thrombocytosis/thrombocytopenia
Thrombophlebitis
Coagulopathies and disseminated intravascular coagulation
Nonbacterial thrombotic endocarditis
139
Q

What are the clinical usefulness of tumour markers?

A

Tumour markers are secreted proteins produced by cancers that are detectable in the serum of patients
Some have found clinical application as a means of monitoring the course of disease and as prognostic factors
Can be used for population screening, diagnosis, prognostic factors, monitoring treatment, diagnosis of relapse and imaging of metastases
The minimal requirements for a tumour marker are:
reliable, quick, cheap assay
high sensitivity (>50%) & specificity (>95%)
high predictive value of positive and negative results

Sensitivity is the percentage of patients with a particular disease who have elevated marker levels and are therefore true positives
Specificity is the percentage of patients without disease who have normal marker levels and are therefore true negatives
Positive predictive value is the percentage of positive results (i.e. elevated marker levels) which are true positives
False-positive rate is the percentage of patients without disease who have an elevated marker level
False-negative rate is the percentage of patients with disease who have a normal marker level

140
Q

What do different tumour markers indicated?

A

hCG is a glycoprotein formed physiologically in the syncytiotrophoblast of the placenta, which is used to diagnose and monitor pregnancy, gestational trophoblastic disease and germ cell tumours
The sensitivity is 100% for testicular and placental choriocarcinomas and hydatidiform moles, 48-86% for NSGCT and 7-14% for seminomas
This is the closest model to a perfect tumour marker

AFP is synthesised by fetal yolk sac, liver, and intestine and in the fetus is the major serum protein acting as an albumin-like carrier protein
Moderately elevated levels are seen in some patients with pancreatic, biliary, gastric, and bronchial cancers, as well as occasional patients with non-malignant hepatic disease where active hepatic regeneration is occurring
Increased levels are found in patients with hepatocellular carcinoma and GCT of the testes, ovary, and midline structures, including mediastinum and pineal gland that contain yolk sac tissue
Serum values occasionally increase as a result of chemotherapy

PLAP (Placental like alkaline phosphate )is an isoenzyme of alkaline phosphatase is elevated in testicular seminoma and ovarian dysgerminoma but slightly elevated serum levels are found in people who smoke

CA19-9 is a mucin found in epithelium of fetal stomach, intestine and pancreas
Its main use is to monitor response to treatment in gastric and pancreatic cancer
Levels do not generally correspond well to tumour bulk though levels above 10,000 mostly indicate the presence of metastatic disease
It is not useful in screening for pancreatic cancer
CA19-9 is eliminated exclusively via bile
Any degree of cholestasis can cause levels to rise
It can be elevated in mucinous tumours of the ovary

Inhibin is a protein secreted by granulosa cells including Sertoli cells and inhibits pituitary FSH secretion
It is a more useful marker than oestradiol in monitoring the rare granulosa cell tumours of the ovary

CEA is a glycoprotein is elevated in carcinomas of the gastrointestinal tract and fetal digestive organs but also in a variety of other malignant and non-malignant conditions
This includes severe benign liver disease, inflammatory lesions (particularly of the gastrointestinal tract), infections, trauma, infarction, collagen diseases, renal impairment, and smoking
Low values are also found in the normal colon
Patients with widespread metastases from tumours, including breast, stomach, bronchus, pancreas, oesophagus, cervix, ovary, and endometrium, may have elevated serum CEA levels, which can be used to monitor the response to treatment

Elevated serum levels of CA 15-3 have been found in 12.5% of women with benign breast disease, in 11% of women with operable breast cancer, and in 64% of women with metastatic breast cancer
The mucin-like antigen has no role in screening because of the low sensitivity for early stages of the disease
It can be elevated in some gynaecological cancers
CA 15-3 elevation increases with increasing stage of disease and highest levels are seen in patients with liver or bone metastases
A rising CA 15-3 level during follow-up can detect relapse 2-9 months before clinical signs or symptoms develop

CA125 is produced from derivatives of the coelomic epithelium, including the pleura, pericardium, peritoneum, fallopian tube, endometrium, and endocervix
It has been detected in many tissues and secretions, but not in the normal ovary
CA 125 is present at the cell surface in more than 80% of non-mucinous epithelial ovarian tumours and in a small percentage of other tumours
However, this is elevated in over 90% of patients with stage III or IV disease but in only 50% with stage I
Over 40% of patients with advanced intra-abdominal cancers of diverse primary site and histology have elevated levels of CA125

CA125 has a low sensitivity for stage I tumours suggesting that it is unlikely to have a role in screening and a clinical trial has failed to demonstrate a survival advantage
Very high CA125 levels prior to surgery are associated with a worse prognosis
However, the most important prognostic factors are the CA125 level after one, two, or three courses of chemotherapy, a short half-life, or greater than 7-fold fall in CA125
It is therefore useful in the monitoring of response to chemotherapy for ovarian cancer and for detection of relapse

SCC is a glycoprotein that is mainly used for monitoring treatment of squamous cell cervical cancer (sensitivity 70-85%) and of head and neck squamous carcinomas (sensitivity 60%)
Elevated levels are found in 17% of all NSCLC and in 31% of squamous NSCLC

PSA is a serine protease produced by prostate epithelium with the function of liquefying the gel which surrounds spermatazoa to enable them to become fully mobile
Elevated serum levels of PSA (>4 ng/ml) occur in 50% of men with intracapsular microscopic prostate cancer and 77% of men with intracapsular macroscopic disease
It is seen in 30-50% of men with benign prostatic hypertrophy
The combination of PSA and digital rectal examination, followed by prostatic ultrasound in patients with abnormal findings, is commonly used for screening in the US but is not recommended in the UK as there is no evidence of survival benefit from early detection of prostate cancer

141
Q

other markers of ovarian cancer?

A

A number of other markers have been found to be elevated in a proportion of patients with ovarian carcinoma

These include
Breast carcinoma-associated mucins such as CASA, OVX1, and HMFG2
Cytokeratin proliferation markers - tissue polypeptide antigen, as well as placental alkaline phosphatase, TATI, CA 19.9, TAG 72.3, LASA, IAP, CSF, ferritin, NB/70K, and galactosyl transferase
None of these markers have yet been shown to be as useful clinically as CA-125
AFP, β-hCG for ovarian germ cell tumours

2-microglobulin: myeloma (60%), NHL (15%)
Neurone-specific enolase: neuroblastoma, SCLC
Paraproteins (monoclonal): myeloma (98%)
Thyroglobulin: papillary and follicular thyroid cancer

142
Q

How are hormones used as tumour markers?

A

Several tumours of endocrine tissue can be diagnosed and treatment monitored by serial measurement
Calcitonin and calcitonin gene-related peptide are used in the screening for medullary carcinoma of the thyroid
Catecholamine metabolites, vanillylmandelic acid and homovanillic acid can be used to detect neuroblastoma

143
Q

How is LDH used as a tumour marker?

A

LDH is often measured as a tumour marker
Measure of the ‘bulk’ of disease
Indicates necrosis as a disease process and therefore associated with tumours that grow rapidly
Non-specific but may identify patients at risk of tumour lysis syndrome

144
Q

How often should tumour markers be checked?

A

For most markers that are useful (PSA, CEA, CA 125) they are measured with each cycle of chemotherapy to assess response
After treatment is complete, they are measured at each clinical review which is most commonly 3 monthly for 2 years and then 6 monthly for a further 3 years
No monitoring once the patient is discharged from surveillance after 5 years

145
Q

What should an ideal tumour marker have?

A

Produced by malignant cells only
Exclusive to a certain malignant type only
Detectable levels at all extent of malignant disease
Specimens can be obtained in a non-invasive manner
Serum levels should equate to disease burden (volume)
Have a short serum half life
Be reproducible and validated